SlideShare a Scribd company logo
TRẠI HÈ HÙNG VƯƠNG LẦN THỨ IX 
CHUYÊN ĐỀ 
PHƯƠNG TÍCH VÀ ỨNG DỤNG 
Nguyễn Quỳnh, Chuyên Hùng Vương, Phú Thọ 
Đào Văn Lương, Chuyên Lào Cai 
Hoàng Thông, Chuyên Lê Quý Đôn, Điện Biên 
HÒA BÌNH, THÁNG 8 NĂM 2013
Trang 1 
MỤC LỤC 
Trang 
Phần A Cơ sở lý thuyết 2 
1. Phương tích của một điểm đối với một đường tròn 2 
2. Trục đẳng phương của hai đường tròn 3 
3. Tâm đẳng phương của ba đường tròn 5 
Phần B Ứng dụng phương tích giải một số bài tập hình học phẳng 7 
1. Các bài tập sử dụng tính chất của phương tích 7 
2. Các bài tập sử dụng tính chất của trục đẳng phương 10 
3. Các bài tập sử dụng tính chất của tâm đẳng phương 23 
Phần C Bài tập đề nghị 28 
1. Đề bài 28 
2. Lời giải 30 
Tài liệu tham khảo 40
Trang 2 
PHẦN A: CƠ SỞ LÝ THUYẾT 
1. Phương tích của một điểm đối với một đường tròn 
1.1 Bài toán 
Cho đường tròn (O; R) và điểm M cố định, OM = d. Một đường thẳng thay đổi qua M cắt 
đường tròn tại hai điểm A và B. Khi đó MA.MB = MO2 − R2 = d 2 − R2. 
Chứng minh 
B 
C 
M 
O 
A 
Gọi C là điểm đối xứng của A qua O. Ta có CB ⊥ AM hay B là hình chiếu của C trên AM. 
    ( OC)(  OA) 
 
Khi đó ta có MA.MB = MA.MB = MC.MA = MO +MO +      
( )( ) 2 2 
= − + = − 
= − = − 
MO OA MO OA MO OA 
2 2 2 2. 
OM OA d R 
1.2 Định nghĩa 
Đại lượng không đổi MA.MB = d 2 − R2 trong Bài toán 1.1 được gọi là phương tích của 
điểm M đối với đường tròn (O), kí hiệu PM/(O). Ta có: 
( ) 
2 2 
/ . M O P = MAMB = d − R . 
1.3 Tính chất 
1.3.1 Tính chất 1 
Điểm M nằm bên ngoài đường tròn (O) khi và chỉ khi /( ) 0. M O P  
Điểm M nằm trên đường tròn (O) khi và chỉ khi /( ) 0. M O P = 
Điểm M nằm bên trong đường tròn (O) khi và chỉ khi /( ) 0. M O P
1.3.2 Tính chất 2 
Trong mặt phẳng, cho đường tròn (O;R) và một điểm M nằm bên ngoài (O). Qua M 
kẻ cát tuyến MAB và tiếp tuyến MT tới (O). Khi đó 
Trang 3 
  
MA.MB = MT 2 = OM2 − R2. 
1.3.3 Tính chất 3 
Cho hai đường thẳng AB,CD phân biệt cắt nhau tại M (M không trùng A,B, C,D). Khi 
    
đó, nếu MA.MB = MC.MD 
thì bốn điểm A,B,C,D cùng nằm trên một đường tròn. 
1.3.4 Tính chất 4 
Cho hai đường thẳng AB,MT phân biệt cắt nhau tại M (M không trùng A,B,T ). Khi 
  
đó, nếu MA.MB = MT 2 
thì đường tròn ngoại tiếp tam giác ABT tiếp xúc với MT tại T. 
1.4 Phương tích trong hệ tọa độ Descartes 
Trong mặt phẳng, với hệ tọa độ Descartes, cho điểm ( ) 0 0 M x ; y và đường tròn 
(C) : x2 + y2 + 2ax + 2by + c = 0. 
Đặt F(x; y) = x2 + y2 + 2ax + 2by + c, khi đó 
P 2 2 
( ) M / O1 
0 0 0 0 0 0 = F(x ; y ) = x + y + 2ax + 2by + c. 
2. Trục đẳng phương của hai đường tròn 
2.1 Định lý và định nghĩa 
Cho hai đường tròn không đồng tâm (O1; R1) và (O2; R2). Tập hợp các điểm M có phương 
tích đối với hai đường tròn bằng nhau là một đường thẳng, đường thẳng này được gọi là 
trục đẳng phương của hai đường tròn (O1) và (O2). 
Chứng minh 
Giả sử điểm M có cùng phương tích đối với hai đường tròn đã cho. 
Gọi H là hình chiếu của M trên O1O2, I là trung điểm của O1O2. Ta có: 
( 2 2 ) ( 2 2 ) 2 2 
⇔ + − + = − 
⇔ − = − 
( )( ) 2 2 
MH HO MH HO R R 
1 2 1 2 
2 2 2 2 
1 2 1 2 . 
HO HO R R 
⇔ HO1 − HO2 HO1 + HO2 = R1 − R2
Trang 4 
M 
I O2 O1 
H 
2 2 
− 
R R 
2 2 1 2 
.2 . 
2 1 1 2 
1 2 
2 
O O HI R R IH 
OO 
⇔ = − ⇔ = 
Do H cố định, suy ra tập hợp các điểm M có cùng phương tích đối với hai đường tròn là 
đường thẳng qua H và vuông góc với O1O2. 
2.2 Tính chất 
Cho hai đường tròn (O1) và (O2). Từ định lý 2.1 ta có các tính chất sau: 
2.2.1 Tính chất 1 
Trục đẳng phương của hai đường tròn vuông góc với đường thẳng nối tâm. 
2.2.2 Tính chất 2 
Nếu hai đường tròn cắt nhau tại A và B thì AB chính là trục đẳng phương của chúng. 
2.2.3 Tính chất 3 
Nếu điểm M có cùng phương tích đối với (O1) và (O2) thì đường thẳng qua M vuông góc 
với O1O2 là trục đẳng phương của hai đường tròn. 
2.2.4 Tính chất 4 
Nếu hai điểm M, N có cùng phương tích đối với hai đường tròn thì đường thẳng MN chính 
là trục đẳng phương của hai đường tròn. 
2.2.5 Tính chất 5 
Nếu 3 điểm có cùng phương tích đối với hai đường tròn thì 3 điểm đó thẳng hàng. 
2.2.6 Tính chất 6
Nếu (O1) và (O2) tiếp xúc nhau tại A thì đường thẳng qua A và vuông góc với O1O2 chính 
là trục đẳng phương của hai đường tròn. 
2.2 Cách xác định trục đẳng phương của hai đường tròn không đồng tâm 
Trong mặt phẳng cho hai đường tròn không đồng tâm (O1) và (O2). Xét các trường hợp sau: 
2.2.1 Trường hợp 1: Hai đường tròn cắt nhau tại hai điểm phân biệt A, B. Khi đó đường 
thẳng AB chính là trục đẳng phương của hai đường tròn. 
2.2.2 Trường hợp 2: Hai đường tròn tiếp xúc nhau tại T. Khi đó tiếp tuyến chung tại T 
chính là trục đẳng phương của hai đường tròn. 
2.2.3 Trường hợp 3: Hai đường tròn không có điểm chung. Dựng đường tròn 3 (O ) cắt 
cả hai đường tròn. Trục đẳng phương của các cặp đường tròn 1 3 (O )và (O ); 2 3 (O )và (O ) 
cắt nhau tại K. Đường thẳng qua K vuông góc với O1O2 là trục đẳng phương của 
Trang 5 
1 2 (O ),(O ). 
2.3 Trục đẳng phương trong Hệ tọa độ Descartes 
Trong mặt phẳng tọa độ Oxy cho các đường tròn không đồng tâm: 
(C ) : x 2 + y 2 
+ 2a x + 2b y + c = 0 , (C ) : x 2 + y 2 
+ 2a + 2b + = 0 
1 1 1 1 2 2 x 2 y c 2 Từ biểu thức phương tích của một điểm đối với một đường tròn trong hệ tọa độ suy ra 
trục đẳng phương của 1 (C ) và 2 (C ) là đường thẳng có phương trình 
( ) ( ) ( ) 1 2 1 2 1 2 2 a − a x + 2 b − b y + c − c = 0 Δ 
3. Tâm đẳng phương của ba đường tròn 
3.1 Định lý và định nghĩa 
Cho 3 đường tròn (C1), (C2) và (C3). Khi đó 3 trục đẳng phương của các cặp đường tròn 
này hoặc trùng nhau hoặc song song hoặc cùng đi qua một điểm. Nếu các trục đẳng 
phương cùng đi qua một điểm thì điểm đó được gọi là tâm đẳng phương của ba đường 
tròn. 
Chứng minh 
Gọi dij là trục đẳng phương của hai đường tròn (Ci) và (Cj). Ta xét hai trường hợp sau. 
a)Giả sử có một cặp đường thẳng song song, không mất tính tổng quát ta giả sử d12 // d23. 
Ta có 12 1 2 23 2 3 d ⊥ OO , d ⊥ O O suy ra 1 2 3 O ,O ,O thẳng hàng. Mà d13 ⊥ O1O3 suy ra 13 23 12 d // d // d
Trang 6 
b)Giả sử d12 và d23 có điểm chung M. Khi đó ta có 
P = M / ( P 
O ) M / 
( O 
) 
 ⇒ = ⇒ ∈ 
 
( ) ( ) 
( ) ( ) 
1 2 
1 3 
2 3 
/ / 13 
/ / 
M O M O 
M O M O 
M d 
=  
P P 
P P 
d12 
d13 
d23 
M 
O1 
O2 
O3 
Từ đây suy ra nếu có hai đường thẳng trùng nhau thì đó cũng là trục đẳng phương của cặp 
đường tròn còn lại. 
Nếu hai trục đẳng phương chỉ cắt nhau tại một điểm thì điểm đó cũng thuộc trục đẳng 
phương còn lại 
3.2 Tính chất 
3.2.1 Tính chất 1: 
Nếu 3 đường tròn đôi một cắt nhau thì các dây cung chung cùng đi qua một điểm 
3.2.2 Tính chất 2: 
Nếu 3 trục đẳng phương song song hoặc trùng nhau thì tâm của 3 đường tròn thẳng hàng. 
3.2.3 Tính chất 3: 
Nếu 3 đường tròn cùng đi qua một điểm và có các tâm thẳng hàng thì các trục đẳng 
phương trùng nhau.
Trang 7 
PHẦN B: ỨNG DỤNG PHƯƠNG TÍCH 
GIẢI MỘT SỐ BÀI TẬP HÌNH HỌC PHẲNG 
1. CÁC BÀI TẬP SỬ DỤNG TÍNH CHẤT CỦA PHƯƠNG TÍCH 
Bài tập 1.1 (S44 Mathematical Reflection MR2-2007) 
Từ một điểm P nằm bên ngoài đường tròn tâm O, kẻ các tiếp tuyến PA, PB tới 
đường tròn (O), (A, B là các tiếp điểm). Gọi M là trung điểm AP và N là giao điểm 
của BM với (O), (N không trùng B). Chứng minh rằng PN = 2MN. 
Lời giải 
Ta có 
A 
P O 
MN.MB = MA2 = MA.MP . Gọi N ' là điểm đối xứng với N qua M, khi đó 
MN.MB = MN '.MB, suy ra MA.MP = MN 'MB, hay tứ giác ABPN ' là tứ giác nội 
tiếp một đường tròn. 
Đặt NAP = α,NAB = β, ta có PAN ' = PBN ' = BAN = β⇒ NAN ' =ANN ' = α + β. 
Mặt khác ANN ' = NAB + NBA = α + β, hay tam giác NPN’ cân tại N suy ra 
PN = 2MN. 
Bài tập 1.2 
N' 
N 
M 
B
Cho đường tròn (O) và hai điểm A, B cố định. Một đường thẳng quay quanh A, cắt (O) tại 
M và N. Chứng minh rằng tâm đường tròn ngoại tiếp tam giác BMN thuộc một đường 
thẳng cố định. 
Lời giải 
Trang 8 
C 
M 
N 
O 
I 
A 
B 
Gọi I là tâm đường tròn ngoại tiếp tam giác MNB. AB cắt (I) tại điểm thứ hai C. Ta có 
A I A O = AC AB = AM AN = (không đổi vì A, (O) cố định). 
/( ) /( ) P . . P 
P 
Suy ra A /( O 
) 
AC 
AB 
= 
Vì A, B cố định và C thuộc AB nên từ hệ thức trên ta có C cố định. 
Suy ra I thuộc đường trung trực của BC cố định. 
Suy ra 
AB.AC 
AK 
= là hằng số nên điểm K cố đinh. Bài tập được chứng minh 
AI 
Bài tập 1.3 
Cho đường tròn (O) và dây AB. Trên tia AB lấy điểm C nằm ngoài đường tròn (O). Từ 
điểm E chính giữa cung lớn AB kẻ đường kính EF cắt dây AB tại D. Tia CE cắt (O) tại 
điểm I. Cho A, B, C cố định, chứng minh rằng khi đường tròn (O) thay đổi nhưng vẫn đi 
qua A, B thì đường FI luôn đi qua một điểm cố định. 
Lời giải
Trang 9 
E 
⇒ = ⇒ điểm K cố định 
A C 
K 
H 
I 
B 
O 
N 
M 
K 
I 
F 
A D 
B C 
Gọi K là giao điểm của FI và AB 
Tứ giác DKIE nội tiếp đường tròn đường kính EK nên 
/[ ] /( ) . . . C EK C O P = CK CD = CI CE = P = CB CA 
CA.CB 
CK 
CD 
Vậy đường thẳng FI luôn đi qua một điểm K cố định 
Bài tập 1.4 
Cho ba điểm cố định A, B, C thẳng hàng (theo thứ tự đó). Một đường tròn (O) thay đổi 
nhưng luôn đi qua B, C. Từ điểm A kẻ các tiếp tuyến AM, AN đến (O). Đường thẳng MN 
cắt AO và AC lần lượt tại H và K. Gọi I là trung điểm BC. Chứng minh rằng đường tròn 
ngoại tiếp tam giác OHI luôn đi qua hai điểm cố định 
Lời giải 
Hiển nhiên điểm I cố định. Do HOIK nội tiếp nên ta có 
[ ] 
2 
/ . . A KO P = AK AI = AH AO = AM 
(do tam giác AMO vuông tại M có MH là đường cao) 
Ta lại có AM là tiếp tuyến của (O) nên ( ) 
AM 2 
= P = AB . AC 
. A / O
Trang 10 
2. CÁC BÀI TẬP SỬ DỤNG TÍNH CHẤT CỦA TRỤC ĐẲNG PHƯƠNG 
Bài tập 2.1 
(IMO 2013 Problem 4) 
Cho tam giác nhọn ABC với trực tâm H. Cho W là một điểm tùy ý trên cạnh BC, khác với 
các điểm B và C. Các điểm M và N tương ứng là chân các đường cao hạ từ B và C. Kí 
hiệu 1 ω là đường tròn ngoại tiếp tam giác BWN, và gọi X là điểm trên 1 ω sao cho WX là 
đường kính của 1 ω . Tương tự, kí hiệu 2 ω là đường tròn ngoại tiếp tam giác CWM, và gọi 
Y là điểm trên 2 ω sao cho WY là đường kính của 2 ω . Chứng minh rằng các điểm X,Y và H 
thẳng hàng. 
Lời giải 
w2 
w1 
Z 
Y 
O2 
X 
O1 
P 
M 
N 
A 
H 
B C 
W 
Gọi P là chân đường cao kẻ từ A của tam giác ABC, O ,O là tâm các đường tròn ω ,ω , 
1 2 1 2 Z là giao điểm thứ hai của ω và ω . 
1 2    
Tứ giác BNMC nội tiếp đường tròn đường kính BC nên AN.AB = AM.AC 
hay A thuộc 
trục đẳng phương của ω và ω . Suy ra A, Z, W cùng nằm trên một đường thẳng vuông 
1 2góc với OO và XY (1) 
1 2       
Tứ giác BNHP nội tiếp nên AH.AP = AN.AB = AZ.AW, 
từ đó PHZW là tứ giác nội tiếp 
hay HZ vuông góc với ZW (2)
Từ (1) và (2) suy ra X, Y, H thẳng hàng, điều phải chứng minh. 
Bài tập 2.2 
Cho tứ giác ABCD nội tiếp đường tròn (O), AB ≠ CD. Dựng hai hình thoi AEDF và 
BMCN có cạnh bằng nhau. Chứng minh bốn điểm E, F, M, N cùng thuộc một đường tròn. 
Lời giải 
Dựng (A,AF) và (B,BM) . 
Do OA = OB và AF = BM nên O nằm trên trục đẳng phương của (A) và (B). 
Trang 11 
M 
N 
E 
O 
A 
D 
C 
B 
F 
Mặt khác, EF, MN lần lượt là trung trực của đoạn AD, BC nên EF ∩MN = {O} 
⇒OE.OF = OM.ON 
Suy ra 4 điểm E, F, M, N cùng thuộc một đường tròn (đpcm). 
Bài tập 2.3 
Cho hai đường tròn ngoài nhau (O1), (O2 ). Kẻ tiếp tuyến chung ngoài A1A2 của hai 
đường tròn (A1∈(O1); A2 ∈(O2 )). Gọi K là trung điểm của A1A2 , từ K kẻ các tiếp tuyến 
KB1,KB2 tới (O1) và (O2 ). A1B1, A2B2 cắt nhau tại L, KL cắt O1O2 tại P . Chứng minh 
rằng B1,B2,P,L cùng nằm trên một đường tròn. 
Lời giải 
Do KA1 = KA2 = KB1 = KB2 nên tứ giác A1B1B2A2 nội tiếp
⇒ LB1.LA1 = LB2.LA2 
Suy ra KL là trục đẳng phương của ( ) O1 và ( ) O2 
⇒ KL ⊥ O1O2 
3 điểm A1,B1,P nhìn đoạn O1K dưới góc 900 nên tứ giác A1B1PK nội tiếp, tương tự tứ 
giác A2B2PK nội tiếp 
Bài tập 2.4 
(IMO 1995) 
Trên đường thẳng d lấy bốn điểm A, B, C, D theo thứ tự đó. Các đường tròn đường kính 
AC và BD cắt nhau tại X và Y. Đường thẳng XY cắt BC tại Z. Trên đường thẳng XY lấy 
một điểm P không trùng với Z, đường thẳng CP cắt đường tròn đường kính AC tại điểm 
thứ hai M, đường thẳng BP cắt đường tròn đường kính BD tại điểm thứ hai N. Chứng 
minh AM, DN, XY đồng quy. 
Lời giải 
Gọi J , J ′,Z lần lượt là giao của AM, DN, AD với XY 
Tứ giác JMCK nội tiếp nên PJ.PK = PM.PC 
Tương tự PJ′.PK = PN.PB 
Do P nằm trên trục đẳng phương của đường tròn đường kính AC và đường tròn đường 
kính BD nên PM.PC = PN.PB⇒ PJ.PK = PJ′.PK hay P ≡ P′ 
Vậy AM, DN, XY đồng quy (đpcm). 
Bài tập 2.5 
Cho tam giác ABC, các đường cao AD, BE, CF đồng quy tại H, M là trung điểm BC, EF 
cắt BC tại I, J là trung điểm của MH. Chứng minh IH vuông góc với OJ. 
Lời giải 
Gọi O, J lần lượt là trung điểm AH, MH. 
Ta có ∠EFD = 2∠HFD = 2∠EBM = ∠EMC 
Suy ra tứ giác FEMD nội tiếp. 
⇒ IE.IF = IM.ID 
Do đó I nằm trên trục đẳng phương của (O,OA) và ( J , JH ) 
Trang 12
⇒IH ⊥ OJ 
Mà OJ là đường trung bình của tam giác AMH nên OJ//AM 
⇒IH ⊥ AM (đpcm) 
Bài tập 2.6 
Cho tam giác ABC nội tiếp trong đường tròn (O). AA’, BB’, CC’ là các đường cao của tam 
giác. Kí hiệu (W ) A là đường tròn đi qua A, A’ và tiếp xúc với OA. (W ),(W ) B C được 
định nghĩa tương tự. Chứng minh rằng ba đường tròn đó cắt nhau tại hai điểm thuộc 
đường thẳng Euler của tam giác ABC. 
Lời giải 
Ta có / (W ) . . / (W ) PH A = HA HA′ = HC HC′ = PH C và do OA, OC lần lượt tiếp xúc với 
W ,W A C nên / (W ) 2 2 / (W ) PO A = OA = OC = PO C 
Suy ra OH là trục đẳng phương của (W ) A và (W ) C 
(W ) (W ) { , } ⇒ A ∩ B = E F ∈HO 
Vậy 3 đường tròn (W ) A , (W ) B , (W ) C cắt nhau tại 2 điểm thuộc đường thẳng Ơ-le của 
tam giác ABC (đpcm). 
Bài tập 2.7 
Cho tam giác không cân ABC nội tiếp đường tròn (O) và ngoại tiếp đường tròn (I). Các 
điểm A’, B’, C’ lần lượt thuộc các đường thẳng BC, CA, AB sao cho 
' ' ' 90o. AIA = BIB = CIC = Chứng minh A’, B’, C’ cùng thuộc một đường thẳng và đường 
thẳng đó vuông góc với OI. 
Lời giải 
Kí hiệu (I, 0) là đường tròn tâm I, bán kính bằng 0. 
Trang 13 
Ta có  
 
 900  
ABC 
IBC = = AIC − = CIA′ 
2 
2 . ⇒ IA′ = A′B A′C hay PA/(I ;0) = PA′/(O) 
Tương tự PB/(I ;0) = PB′/(O) , PC/(I ;0) = PC'/(O)
Suy ra A′,B′,C′ cùng thuộc trục đẳng phương của (O) và (I,0), đường thẳng này vuông 
góc với OI (đpcm). 
Bài tập 2.8 
Cho đường tròn tâm O đường kính AB, và điểm H cố định thuộc AB. Từ điểm K thay đổi 
trên tiếp tuyến tại B của O, vẽ đường tròn (K; KH) cắt (O) tại C và D. Chứng minh rằng 
CD luôn đi qua một điểm cố định. 
Lời giải 
Trang 14 
I 
M 
D 
C 
B 
A O 
K 
H 
Gọi I là điểm đối xứng của H qua B, suy ra I cố định và thuộc (K). 
Gọi M là giao điểm của CD và AB. 
Vì CD là trục đẳng phương của (O) và (K) nên ta có: 
= = 
. . . 
MH MI MC MD MAMB 
MB BH MB BI MB MB BA 
( )( ) ( ) 
( )( ) 2 
⇔ + + = + 
⇔ + − = + 
⇔ − = + 
⇔ = 
MB BH MB BH MB MB BA 
2 2 2 
2 
. 
. 
MB BH MB MB BA 
BH 
BM 
BA 
Vì A, B, H cố định suy ra M cố định.
Bài tập 2.9 
Cho tam giác ABC có đỉnh A cố định và B, C thay đổi trên đường thẳng d cố định sao cho 
nếu gọi A’ là hình chiếu của A lên d thì A′B.A′C âm và không đổi. Gọi M là hình chiếu 
của A’ lên AB; N là hình chiếu của A’ lên AC; K là giao điểm của các tiếp tuyến của 
đường tròn ngoại tiếp tam giác A’MN tại M và N. Chứng minh rằng K thuộc một đường 
thẳng cố định. 
Lời giải 
Trang 15 
I 
P 
D 
H 
K 
M 
N 
A 
B A' C 
Gọi O là tâm đường tròn ngoại tiếp tam giác A’MN và I là giao điểm của OK và MN. Ta 
thấy O chính là trung điểm của AA’. 
Dễ thấy 2 
AM.AB = AA′ = AN.AC 
Suy ra tứ giác BMNC nội tiếp. 
  ⇒ AMN = ACB 
Mà   ADB = ACBNên   AMN = ADB 
Suy ra MPDB nội tiếp. 
Do đó ta có 2 
AP.AD = AM.AB = AA′ Mà A, A’ và D cố định suy ra P cố định. 
1 
Gọi H là hình chiếu của K trên AA’.Ta có AP . AH = AI . 
AK = IN 2 = AA′ 
2 4 
Mà A, P, A’ cố định suy ra H cố định. 
Vậy K thuộc đường thẳng qua H và vuông góc với AA’. 
Bài tập 2.10 
Trên đường thẳng d lấy 4 điểm A, B, C, D (theo thứ tự đó). Đường tròn đường kính AC và 
BD cắt nhau tại X, Y. Đường thẳng XY cắt BC tại Z. Lấy P là một điểm trên XY khác Z.
Đường thẳng CP cắt đường tròn đường kính AC tại điểm thứ hai là M, và BP cắt đường 
tròn đường kính BD tại điểm thứ hai là N. Chứng minh rằng AM, DN và XY đồng qui. 
Lời giải 
Gọi Q, Q’ lần lượt là giao điểm của DN và AM với XY. Ta cần chứng minh Q ≡ Q′ . 
Tứ giác QMCZ nội tiếp, suy ra PM.PC = PQ.PZ 
Tứ giác NQ’ZB nội tiếp, suy ra PQ′.PZ = PN.PB 
Mà P thuộc XY là trục đẳng phương của đường tròn đường kính AC và đường tròn đường 
kính BD nên PN.PB = PX.PY = PM.PC 
Trang 16 
Q 
P 
X 
A B Z 
C D 
N 
M 
Y 
Suy ra PQ.PZ = PQ′.PZ ⇒Q ≡ Q′ 
Vậy XY, AM và DN đồng quy. 
Bài tập 2.11 
Cho H là trực tâm tam giác ABC không cân góc A nhọn; Hình chiếu vuông góc của H trên 
AB, AC theo thứ tự là E, F. Gọi D là trung điểm BC; P, Q là giao điểm của hai đường tròn 
đường kính AD và BC. Chứng minh H, P, Q thẳng hàng và các đường thẳng BC, EF, PQ 
đồng quy. 
Lời giải 
Gọi G là chân đường cao kẻ từ đỉnh A của tam giác ABC. 
Ta có /[ ] /[ ] . . H BC H AD P = HE HC = HG HA = P suy ra H nằm trên trục đẳng phương của hai 
đường tròn đường kính BC và AD. Suy ra H, P, Q thẳng hàng.
Gọi I là giao điểm của EF và BC. (DEF) là đường tròn Euler của tam giác ABC nên G 
nằm trên (DEF). Do đó /[ ] /[ ] . . . I BC I AD P = IB IC = IE IF = IG ID = P Suy ra I, P, G thẳng 
hàng hay BC, EF, PQ đồng quy tại I. 
Bài tập 2.12 
Cho tam giác ABC có trực tâm H. Đường tròn đi qua B, C cắt AB, AC tại D, E. Gọi F là 
trực tâm tam giác ADE và I là giao điểm của BE và CD. Chứng minh I, H, F thẳng hàng 
Lời giải 
Gọi F1, F2 là hình chiếu vuông góc của F lên AB, AC; H1,H2 là hình chiếu vuông góc của 
H lên AB, AC 
Ta có 1 2 1 2 FF.FE = FF .FD; HH .HC = HH .HB (tính chất trực tâm) 
Mặt khác ta lại có IE.IB = IC.ID 
Suy ra F, H, I cùng thuộc trục đẳng phương của hai đường tròn đường kính BD và CE nên 
chúng thẳng hàng 
Bài tập 2.13 
Cho tứ giác ABCD có các cạnh AB và CD cắt nhau tại I, AD và BC cắt nhau tại K. 
1) Chứng minh rằng trực tâm của các tam giác AID, ABK, BCI, CDK thẳng hàng 
2) Chứng minh rằng trung điểm của các đoạn AC, BD, IK thẳng hàng. 
Lời giải 
1)Gọi O1; O2; O3 lần lượt là trung điểm các đoạn AC, BD, IK 
Các đường cao của tam giác CDK lần lượt là CC', DD'; KK' và H là trực tâm tam giác 
Trang 17
Trang 18 
K' 
A C' 
K 
D' 
H 
O1 
O2 
O3 
I 
D 
B 
C 
Ta có ( ) ( ) / 1 / 2 . '; . ' H O H O P = HC HC P = HD HD xét đường tròn đường kính CD ta lại có 
HC.HC' = HD.HD' nên H thuộc trục đẳng phương của hai đường tròn (O1),(O2) 
Chứng minh tương tự cho trực tâm các tam giác AID, ABK, BCI cũng có cùng phương 
tích với hai đường tròn (O1),(O2). Suy ra điều phải chứng minh 
2)Tương tự ta chứng minh được H có cùng phương tích với hai đường tròn (O2) và (O3) 
Gọi J là trực tâm của tam giác BCI. Khi đó ta cũng chứng minh được J có cùng phương 
tích với ba đường tròn (O1),(O2), (O3). Suy ra ba đường tròn này đôi một nhận HJ làm 
trục đẳng phương suy ra đpcm 
Bài tập 2.14 
Cho đường tròn tâm O và hai đường kính AB, CD. Tiếp tuyến với (O) tại B cắt AC tại P, 
PD cắt (O) tại điểm thứ hai G. Chứng minh rằng AG, BC, PO đồng quy. 
Lời giải 
Gọi I, I' lần lượt là trung điểm PA, PC. Ta có bốn điểm O, B, C, I thuộc đường tròn (T) 
đường kính BI 
Ta chứng minh được GAI ' = CDG = COI ' = GOI ' suy ra bốn điểm O, A, G, I' cùng 
thuộc đường tròn (T') khi đó /( ) /( ') 0 O T O T P = P = 
Hơn nữa /( ) /( ') . . ' P T P T P = PI PC = PA PI = P
Trang 19 
j 
F 
I 
O 
H 
I I' 
E 
G 
P 
N M 
D 
A 
B C 
B 
C 
O 
A 
D 
suy ra OP là trục đẳng phương của (T) và (T') 
Nhưng AG là trục đẳng phương của (O) và (T'), BC là trục đẳng phương của (O) và (T) 
Vậy AG, BC, PO đồng quy. 
Bài tập 2.15 
Cho tam giác ABC có đường cao BD và CE cắt nhau tai H. M là trung điểm của BC, N là 
giao điểm của DE và BC. Chứng minh rằng NH vuông góc với AM. 
Lời giải 
Ta có 
    
= = = 
DEH DAH DBC FEH 
 2. 2.  
⇒ = = = 
FED FEH DBC DMC 
Suy ra tứ giác EDMF nội tiếp. 
Từ đó ta có NE.ND = NF.NM , suy ra 
N nằm trên trục đẳng phương của đường 
tròn đường kính MH và đường tròn 
đường kính AH. 
Mặt khác H là giao điểm của (O) và (I), 
suy ra NH chính là trục đẳng phương của (O) và (I). 
Suy ra NH ⊥ OI , rõ ràng OI // AM, do đó NH ⊥ AM . 
Bài tập 2.16 
Cho tam giác ABC. Một đường thẳng song song với BC cắt AB, AC tại D và E. Gọi P là 
một điểm bên trong tam giác ADE, F và G là giao của DE với BP và CP. Đường tròn tâm
(O) ngoại tiếp tam giác PDG, đường tròn tâm (I) ngoại tiếp tam giác PEF cắt nhau tại 
điểm thứ hai là Q. Chứng minh rằng AQ ⊥ OI 
Lời giải 
Gọi M là giao điểm thứ hai của AB và (PDG), N là giao thứ hai của AC và (PFG) 
Trang 20 
M 
N 
P 
F G 
E 
A 
D 
B C 
Ta có   AMP = PGD và   PGD = PCB (đồng vị), suy ra   AMP = PCB , suy ra BMPC nội tiếp. 
Chứng minh tương tự PNCB nội tiếp. Suy ra BMNC nội tiếp, suy ra AM.AB = AN.AC 
Mà AD AE 
= (Định lý Thalet). Suy ra AM.AD = AN.AE 
AB AC 
Do đó A thuộc trục đẳng phương PQ của (PDG) và (PEF) suy ra AQ ⊥ OI . 
Bài tập 2.17 
Cho tam giác ABC không cân nội tiếp (O) và ngoại tiếp (I). Các đường thẳng qua I vuông 
góc với AI, BI, CI cắt BC, CA, AB tại M, N, P theo thứ tự. Chứng minh M, N, P cùng nằm 
trên một đường thẳng vuông góc với OI. 
Lời giải
Trang 21 
Giả sử AB  AC khi đó M nằm trên tia đối của tia BC. Ta có 
C C 
 AIB = 90 
0 + ⇒MIB  = = MCI 
 2 2 
Suy ra ΔMIB ∼ ΔMCI ⇒MI 2 = MB.MC 
Suy ra M nằm trên trục đẳng phương của (O) và đường tròn tâm I. Tương tự với N, P ta 
suy ra M, N, P thẳng hàng và đường thẳng đi qua ba điểm đó vuông góc với OI. 
Bài tập 2.18 
Cho tam giác ABC nhọn. Các đường cao BE, CF cắt nhau tại H. Trên các tia FB, EC lấy 
các điểm P, Q theo thứ tự sao cho FP = FC, EQ = EB. PQ cắt CP tại K, I, J theo thứ tự tà 
trung điểm BQ, CP. Chứng minh HK vuông góc với IJ. 
Lời giải 
Từ giả thiết ta có BPC = BQC = 450 
suy ra tứ giác BCQP nội tiếp /[ ] /[ ] . . K BQ K CP ⇒ P = KB KQ = KC KP = P 
Theo tính chất trực tâm tam giác tac có 
/[ ] /[ ] . . H BQ H CP P = HB HE = HC HF = P 
vậy HK là trục đẳng phương của hai đường tròn đường kính 
BQ và CP và IJ là đường nối tâm của hai đường nối tâm của hai đường tròn nên HK ⊥ IJ
Bài tập 2.19 
Cho hai đường tròn ngoài nhau (O1), (O2 ). Kẻ các tiếp tuyến chung ngoài A1A2 và chung 
trong B1B2 của hai đường tròn (A1,B1∈(O1); A2,B2 ∈(O2 )). Chứng minh A1B1, A2B2, 
O1O2 đồng quy. 
Lời giải 
Gọi M là giao điểm của A1B1 với A2B2 . Dễ dàng có A1B1 ⊥ A2B2 
Gọi (C1),(C2 ) lần lượt là các đường tròn đường kính A1A2 , B1B2 
Do 0 
∠A1MA2 = ∠B1MB2 = 90 nên M nằm trên trục đẳng phương của (C1) và (C2 ) . 
O1A1 = O1B1 và O1A1,O1B1 lần lượt là tiếp tuyến của (C1),(C2 ) nên O1 nằm 
Trang 22 
Mặt khác 2 2 
trên trục đẳng phương của (C1) và (C2 ) . 
Tương tự O2 cũng nằm trên trục đẳng phương của (C1) và (C2 ) 
Suy ra O1,M,O2 thẳng hàng.
3. CÁC BÀI TẬP SỬ DỤNG TÍNH CHẤT CỦA TÂM ĐẲNG PHƯƠNG 
Bài tập 3.1 
Cho đường tròn (O) đường kính AB và CD. Tiếp tuyến tại B của (O) cắt AC tại E, DE cắt 
(O) tại điểm thứ hai F. Chứng minh rằng AF, BC, OE đồng quy. 
Lời giải 
Trang 23 
F 
E 
B 
D 
O 
A 
C 
Kí hiệu (C1),(C2 ) lần lượt là đường tròn ngoại tiếp tam giác AEF, BCE. 
Ta có AF, BC là trục đẳng phương của (O) và (C1), (O) và (C2 ) . 
Mặt khác OAF = FDB = FEA,OBC = CEB 
Suy ra OA, OB lần lượt là tiếp tuyến của (C1),(C2 ) và lại có 2 2 
OA = OB 
Do đó OE là trục đẳng phương của (C1) và (C2 ) . Suy ra AF, BC, OE đồng quy tại tâm 
đẳng phương của ba đường tròn. 
Bài tập 3.2 
(Iran MO 1996) 
Trên hai cạnh AB, AC của tam giác ABC lần lượt lấy các điểm D, E sao cho DE song song 
với BC. Gọi P là điểm tùy ý bên trong tam giác ABC, các đường thẳng PB, PC cắt DE tại 
F và G. Gọi 1 2 O ,O lần lượt là tâm các đường tròn ngoại tiếp các tam giác PDG, PEF. 
Chứng minh rằng AP vuông góc với 1 2OO .
Trang 24 
Lời giải 
N 
M 
F G 
O2 O1 
E 
A 
B 
C 
P 
D 
Gọi M là giao điểm thứ hai của AB với (O1), N là giao điểm thứ hai của AC với (O2 ) 
Suy ra tứ giác B, P, C, M nằm trên một đường tròn, tương tự B, P, C, N nằm trên một 
đường tròn do đó tứ giác BMNC nội tiếp. Mà DE//BC ta thu được tứ giác MDEN nội tiếp. 
Áp dụng định lý về tâm đẳng phương cho các đường tròn ngoại tiếp DGP, PEF và DENM 
ta có DM ∩EN = {A} nằm trên trục đẳng phương của (O1) và (O2 ) 
Suy ra AP ⊥ O1O2 (đpcm). 
Bài tập 3.3 
Cho nửa đường tròn đường kính AB và một diểm C nằm trên nửa đường tròn đó. Gọi H là 
hình chiếu của C trên AB. Đường tròn đường kính CH cắt CA tại E, CB Tại F và đường 
tròn đường kính AB tại D. Chứng minh rằng CD, EF, AB đồng quy. 
Lời giải 
Do ACB = 900 nên EF là đường kính của đường tròn đường kính CH 
⇒CEF =ACH = CBA 
Suy ra tứ giác AEFB nội tiếp.
Áp dụng định lý về tâm đẳng phương cho đường tròn ngoại tiếp tứ giác AEFB, đường 
tròn đường kính AB và đường kính EF ta có CD, EF, AB đồng quy (đpcm). 
Bài tập 3.4 
Cho tam giác ABC nội tiếp đường tròn ( );( ),( ) O I Ia lần lượt là đường tròn nội tiếp và 
bàng tiếp góc A của tam giác ABC. Giả sử IIa cắt BC và (O) lần lượt tại A′ ,M . Gọi N là 
trung điểm cung MBA. , NI NIa cắt (O) lần lượt tại S,T. Chứng minh rằng S,T, A′ thẳng 
hàng. 
Lời giải 
Ta có: 
Trang 25 
 (  ) (  )  1 1 
NST = sd NA + sd AS = sd NM + sd AS = NIM 
2 2 
Suy ra   
IaTS = IaIS 
Suy ra tứ giác IaTI nội tiếp đường tròn (w1) 
Mặt khác   900 IBIa = ICIa = nên tứ giác IBIaC nội tiếp đường tròn (w2 ). 
Ta thấy IIa là trục đẳng phương của (w1) và (w2 ), BC là trục đẳng phương của (O) và 
(w2 ), TS là trục đẳng phương của (O) và (w1) . 
Theo định lý về tâm đẳng phương thì , , IIa TS BC đồng quy tại A′ . 
Vậy T, A′,S thẳng hàng (đpcm). 
Bài tập 3.5 
Bên ngoài tam giác ABC vẽ các tam giác cân BCD, CAE, ABF có các cạnh đáy tương ứng 
là BC, CA, AB. Chứng minh rằng các đường thẳng lần lượt đi qua A, B, C và vuông góc 
với EF, FD, DE tương ứng đồng quy. 
Lời giải 
Gọi C1,C2,C3 lần lượt là đường tròn tâm D, bán kính DB, đường tròn tâm E, bán kính 
EA, đường tròn tâm F bán kính FA.
Đường thẳng qua C vuông góc với DE là trục đẳng phương của (C1) và (C2 ), đường 
thẳng qua A vuông góc với EF là trục đẳng phương của (C2 ) và (C3 ) , đường thẳng qua 
B vuông góc với DF là trục đẳng phương của (C1) và (C3 ) . 
Do đó 3 đường thẳng này đồng quy tại tâm đẳng phương của 3 đường tròn (đpcm). 
Bài tập 3.6 
Cho tam giác ABC, các điểm A’, B’ lần lượt nằm trên hai cạnh BC và CA. Chứng minh 
rằng trục đẳng phương của hai đường tròn đường kính AA’ và BB’ đi qua trực tâm H của 
tam giác ABC. 
Lời giải 
Kí hiệu ( ) ( ) 1 2 O , O lần lượt là đường tròn đường kính AA′ và BB′ . Gọi M là trung điểm 
AB. 
Dễ thấy (M,MA)∩(O1) = {A,E},(M,MA)∩(O2 ) = {B,F} (E, F lần lượt là chân đường 
cao hạ từ A, B của tam giác ABC). 
Do AE, BF giao nhau tại trực tâm H và áp dụng định lý về tâm đẳng phương ta suy ra trục 
đẳng phương của (O1) và (O2 ) đi qua H (đpcm). 
Bài tập 3.7 
Cho đường tròn tâm O đường kính AB . Một điểm H thuộc đoạn AB. Đường thẳng qua H 
vuông góc với AB cắt đường tròn tại C. Đường tròn đường kính CH cắt AC, BC và (O) lần 
lượt tại D, E và F. 
a) Chứng minh rằng AB, DE và CF đồng quy. 
b) Đường tròn tâm C bán kính CH cắt (O) tại P và Q. Chứng minh rằng P, D, E, Q thẳng 
hàng. 
Lời giải 
a) Ta có 2 
Trang 26 
P 
Q 
M 
E 
D 
C 
A O H B 
CA.CD = CH = CB.CE , suy ra ADEB nội tiếp. 
Xét các đường tròn (ADEB), (O) và đường tròn đường 
kính CH, thì DE, AB và CF lần lượt là các trục đẳng 
phương của các cặp đường tròn trên nên chúng đồng quy. 
b) Ta có PQ là trục đẳng phương của
(C) và (O) nên OC ⊥ PQ . Ta cũng dễ thấy OD ⊥ DE . 
Hơn nữa M chính là tâm đẳng phương của ba đường tròn (O), ( C) và đường tròn đường 
kính CH. Suy ra PQ đi qua M. 
Vậy DE, PQ cùng đi qua M và cùng vuông góc với OC nên trùng nhau. Hay D, E, P, Q 
thẳng hàng. 
Trang 27
Trang 28 
PHẦN C: BÀI TẬP ĐỀ NGHỊ 
1. ĐỀ BÀI 
Bài tập 1 
Cho tam giác ABC, trực tâm H. M 
1, M 
2, M 
3 lần lượt là trung điểm BC, CA, AB. 
(M 
1, M 
1H) ∩ BC={A 
1, A 
2}, (M 
2, M 
2H) ∩ AC={B 
1, B 
2}, (M 
3, M 
3H) ∩ AB={C 
1, C 
2}, 
CMR: A 
1, A 
2 B 
1, B 
2, C 
1, C 
2 cùng thuộc một đường tròn. 
Bài tập 2 
Cho hình thang ABCD (ABCD). K, L là hai điểm trên AB, CD sao cho 
AK 
BK 
= 
DL 
CL 
. Giả 
sử P, Q nằm trên đoạn thẳng KL sao cho ∠APB =∠BCD và ∠CQD=∠ABC. CMR bốn 
điểm P, Q, B, C cùng thuộc một đường tròn. 
Bài tập 3 
Cho tứ giác ABCD có cạnh AB và CD cắt nhau tại I, AD và BC cắt nhau tại K. 
a) CMR trực tâm của các tam giác AID, ABK, BCI, CDK thẳng hàng. 
b) CMR trung điểm của các đoạn thẳng AC, BD, IK thẳng hàng. 
Bài tập 4 
Cho tam giác ABC có các đường phân giác (trong và ngoài) tại các đỉnh A, B, C theo thứ 
tự cắt các cạnh đối tại D và D’, E và E’, K và K’. Chứng minh rằng trung điểm của các 
đoạn thẳng DD’, EE’, KK’ thẳng hàng. 
Bài tập 5 
Cho tứ giác lồi ABCD. Qua đỉnh D của tứ giác kẻ các đường thẳng a, b, c sao cho a ⊥ 
DA, b⊥ DB, c⊥ DC. Các đường thẳng a, b, c này theo thứ tự cắt các đường thẳng BC, 
CA, AB tại A’, B’, C’. CMR ba điểm A’, B’, C’ thẳng hàng. 
Bài tập 6 
Cho tam giác ABC. Các phân giác ngoài góc A, B, C lần lượt cắt cạnh đối diện tại 
A1, B1, C1. CMR A1, B1, C1 thẳng hàng và nằm trên đường vuông góc với đường thẳng 
nối tâm đường tròn nội tiếp và tâm đường tròn ngoại tiếp tam giác ABC. 
Bài tập 7
Cho tam giác ABC nội tiếp (O). (I), (Ia) lần lượt là đường tròn nội tiếp và bàng tiếp góc 
A. Giả sử I Ia giao BC và (O) lần lượt tại A’, M. Gọi N là trung điểm cung MBA. NI, NIa 
giao (O) lần lượt tại S,T. CMR S, T, A’ thẳng hàng. 
Bài tập 8 (Định lý Brianchon) 
Cho lục giác ABCDEF ngoại tiếp (O). CMR AD, BE, CF đồng quy. 
Bài tập 9 
Cho tam giác ABC và đường thẳng d. Gọi A’, B’, C’ lần lượt là hình chiếu của A, B, C 
trên d. Gọi d 
1, d 
2, d 
3 theo thứ tự là các đường thẳng đi qua A’, B’, C’ và vuông góc với 
Trang 29 
BC, CA, AB. Chứng minh rằng các đường thẳng d 
1, d 
2, d 
3 đồng quy. 
Bài tập 10 
Cho (O,R) và hai điểm P,Q cố định (P nằm ngoài (O), Q nằm trong (O)). Dây cung AB 
của (O) luôn đi qua Q. PA, PB lần lượt giao (O) lần thứ hai tại D,C. Chứng minh rằng CD 
luôn đi qua một điểm cố định. 
Bài tập 11 
Cho (O1,R1) tiếp xúc ngoài với (O2,R2) tại M (R2R1). Xét điểm A di động trên đường 
tròn sao cho A,O1,O2 không thẳng hàng. Từ A kẻ tiếp tuyến AB, AC tới (O 
1). Các đường 
thẳng MB, MC cắt tại (O 
2) tại E, F. D là giao điểm của EF với tiếp tuyến tại A của (O 
2). 
Chứng minh rằng D di động trên một đường thẳng cố định. 
Bài tập 12 
Cho tam giác ABC có D là trung điểm của cạnh BC. Gọi d là đường thẳng qua D và 
vuông góc với đường thẳng AD. Trên đường thẳng d lấy một điểm M bất kì. Gọi E, F lần 
lượt là trung điểm của các đoạn thẳng MB, MC. Đường thẳng qua E vuông góc với d cắt 
đường thẳng AB tại P, đường thẳng qua F vuông góc với d cắt đường thẳng AC tại Q. 
Chứng minh rằng đường thẳng qua M, vuông góc với đường thẳng PQ luôn đi qua một 
điểm cố định khi M di động trên đường thẳng d.
Trang 30 
2. LỜI GIẢI 
Bài tập 1 
Cho tam giác ABC, trực tâm H. M 
1, M 
2, M 
3 lần lượt là trung điểm BC, CA, AB. 
(M 
1, M 
1H) ∩ BC={A 
1, A 
2}, (M 
2, M 
2H) ∩ AC={B 
1, B 
2}, (M 
3, M 
3H) ∩ AB={C 
1, C 
2}. 
CMR: A 
1, A 
2, B 
1, B 
2, C 
1, C 
2 cùng thuộc một đường tròn. 
Lời giải 
Do M 
1M 
2 // AB và AB⊥HC nên M 
1M 
2⊥HC 
Suy ra HC là trục đẳng phương của (M 
1) và (M 
2). 
⇒ CA 
1 . CA 
2 = CB 
1 . CB 
2 
Suy ra A 
1, A 
2, B 
1, B 
2 thuộc đường tròn (W 
1). 
Tương tự A 
1, A 
2,C 
1, C 
2 thuộc đường tròn (W 
2), C 
1, C 
2, B 
1, B 
2 thuộc đường tròn (W 
3). 
Nếu 6 điểm A 
1, A 
2, B 
1, B 
2, C 
1, C 
2 không cùng thuộc một đường tròn thì các trục đẳng 
phương của 3 đường tròn (W 
1), (W 
2), (W 
3) phải đồng quy tại một điểm, nhưng chúng lại 
cắt nhau tại A, B, C nên vô lý. Vậy ta có đpcm. 
Bài tập 2 
Cho hình thang ABCD (ABCD). K, L là hai điểm trên AB, CD sao cho 
AK 
BK 
= 
DL 
CL 
. Giả 
sử P, Q nằm trên đoạn thẳng KL sao cho ∠APB =∠BCD và ∠CQD=∠ABC. CMR bốn 
điểm P, Q, B, C cùng thuộc một đường tròn. 
Lời giải 
Từ giả thiết, 
AK 
BK 
= 
DL 
CL 
suy ra AD, BC, KL đồng quy tại E. 
Dựng đường tròn (O1) đi qua hai điểm C, D và tiếp xúc với BC, (O2) đi qua hai điểm A; 
B và tiếp xúc với BC. Khi đó ∠DQC = ∠ABC=∠DCE nên Q∈(O1), tương tự P∈(O2).
Trang 31 
Gọi F là giao điểm thứ hai của EQ với (O1). Ta có: 
EF . EQ = EC 2 (1) 
Mặt khác, dễ dàng có ∠O1CD=∠O2BA do đó △ AO2B ~ △ DO 
1C 
⇒ 
O 
1C 
O 
2B 
= 
DC 
AB 
= 
EC 
EB 
=k ⇒ E, O 
1, O 
2 thẳng hàng và 
EO 
1 
EO 
2 
=k ⇒ 
→ 
EO 
1 = k 
→ 
EO 
2 
Suy ra phép vị tự H(E,k): (O1)→ (O2). Mà E, F, P thẳng hàng, F∈ (O1), P∈ (O2) nên 
→ 
EF =k 
→ 
EP ⇒ 
EF 
EP 
=k= 
EC 
EB 
(2) 
Từ (1), (2) suy ra EP . EQ = EC . EB 
Vậy 4 điểm P, Q, B, C cùng thuộc một đường tròn (đpcm). 
Bài tập 3 
Cho tứ giác ABCD có cạnh AB và CD cắt nhau tại I, AD và BC cắt nhau tại K. 
a) CMR trực tâm của các tam giác AID, ABK, BCI, CDK thẳng hàng. 
b) CMR trung điểm của các đoạn thẳng AC, BD, IK thẳng hàng. 
Lời giải 
a) Gọi O 
1, O 
2, O3 theo thứ tự là trung điểm của AC, BD, IK. Các đường cao của tam giác 
CDK lần lượt là CC’, DD’, KK’ và H là trực tâm. 
Kí hiệu Γ 
1, Γ 
2, Γ 
3 theo thứ tự là các đường tròn tâm O 
1 bán kính O 
1A, tâm O 
2 bán kính 
O 
2B, tâm O 
3 bán kính O 
3K. 
Ta có phương tích của điểm H đối với đường tròn Γ 
1 và Γ 
2 là P 
(H/Γ 
1)= HC . HC’ 
P 
(H/Γ 
2) = HD . HD’ . Xét đường tròn đường kính CD có HC . HC’ = HD . HD’ 
Do đó H thuộc trục đẳng phương của hai đường tròn Γ 
1 và Γ 
2.
Chứng minh tương tự, trực tâm các tam giác AID, ABK, BCI cũng có cùng phương tích 
đối với hai đường tròn Γ 
1 và Γ 
2. Vậy trực tâm của các tam giác AID, ABK, BCI, CDK 
Trang 32 
thẳng hàng ( điều cần chứng minh) 
b) O 
3 là trung điểm IK, ta cũng chứng minh được P 
(H/Γ 
2) = P 
(H/Γ 
3). 
Xét trực tâm J của tam giác BCI, khi đó P 
(J/Γ 
1)= P 
(J/Γ 
2)= P 
(J/Γ 
3) suy ra ba đường tròn 
này đôi một nhận đường thẳng HJ làm trục đẳng phương, vì vậy O 
1, O 
2 và O 
3 thẳng hàng 
(đpcm) . 
Lưu ý: HJ⊥ O 
1O 
3. 
Bài tập 4 
Cho tam giác ABC có các đường phân giác (trong và ngoài) tại các đỉnh A, B, C theo thứ 
tự cắt các cạnh đối tại D và D’, E và E’, K và K’. Chứng minh rằng trung điểm của các 
đoạn thẳng DD’, EE’, KK’ thẳng hàng.
Trang 33 
Lời giải 
Gọi O 
1, O 
2, O 
3 theo thứ tự là trung điểm DD’, EE’, KK’ ; gọi Γ 
1, Γ 
2, Γ 
3 theo thứ tự là các 
đường tròn tâm O 
1 bán kính O 
1D, tâm O 
2 bán kính O 
2E, tâm O 
3 bán kính O 
3K. Giả sử Γ là 
đường tròn ngoại tiếp tam giác ABC có tâm O bán kính R. Vì ΔO 
1BA ∽ ΔO 
1AC nên 
O 
1B.O 
1C = O 
1A2 
= O 
1D2 
, từ đó: 
P 
(O 
1/Γ) = O 
1O2 
- R2 
= O 
1B . O 
1C = O 
1D2 
Nhưng P 
(O/Γ 
1) = OO 
2 
- O 
1 
1D2 
, suy ra: P 
(O/Γ 
1) = R2 
Chứng minh tương tự ta có: P 
(O/Γ 
2) = P 
(O/Γ 
3) = R2 
Mặt khác nếu gọi H là trực tâm tam giác DEK, tương tự như ví dụ 3 ta chứng minh được: 
P 
(H/Γ 
1) = P 
(H/Γ 
2) = P 
(H/Γ 
3) 
Vậy ba đường tròn Γ 
1, Γ 
2, Γ 
3 đôi một nhận đường thẳng OH làm trục đẳng phương do đó 
O 
1, O 
2, O 
3 thẳng hàng. 
Bài tập 5 
Cho tứ giác lồi ABCD. Qua đỉnh D của tứ giác kẻ các đường thẳng a, b, c sao cho a ⊥ 
DA, b⊥ DB, c⊥ DC. Các đường thẳng a, b, c này theo thứ tự cắt các đường thẳng BC, 
CA, AB tại A’, B’, C’. CMR ba điểm A’, B’, C’ thẳng hàng. 
Lời giải 
Gọi trung điểm của AA’, BB’, CC’ theo thứ tự là O 
1, O 
2, O 
3 ; gọi Γ 
1, Γ 
2, Γ 
3 theo thứ tự là 
các đường tròn tâm O 
1 bán kính O 
1A, tâm O 
2 bán kính O 
2B, tâm O 
3 bán kính O 
3C, thế thì 
P 
(D/Γ 
1) = P 
(D/Γ 
2) = P 
(D/Γ 
3) = 0.
Trang 34 
Mặt khác, nếu gọi H là trực tâm tam giác ABC thì tương tự ví du 3 ta có 
P 
(H/Γ 
1) = P 
(H/Γ 
2) = P 
(H/Γ 
3) . Từ đó ba điểm O 
1, O 
2, O 
3 thẳng hàng. 
Gọi P, Q, R theo thứ tự là trung điểm của BC, CA, AB. Sử dụng định lí Menelaus cho tam 
giác PQR và ba điểm thẳng hàng O 
1, O 
2, O 
3, ta có: 
O 
1Q 
O 
1R 
. 
O 
2R 
O 
2P 
. 
O 
3P 
O 
3Q 
= 1. 
Nhưng 
O 
1Q 
O 
1R 
= 
A’C 
A’B 
; 
O 
2R 
O 
2P 
= 
B’A 
B’C 
; 
O 
3P 
O 
3Q 
= 
C’B 
C’A 
. 
Áp dụng định lí Menelaus đảo cho tam giác ABC, ta có A’, B’, C’ thẳng hàng. 
Bài tập 6 
Cho tam giác ABC. Các phân giác ngoài góc A, B, C lần lượt cắt cạnh đối diện tại 
A1, B1, C1. CMR A1, B1, C1 thẳng hàng và nằm trên đường vuông góc với đường thẳng 
nối tâm đường tròn nội tiếp và tâm đường tròn ngoại tiếp tam giác ABC. 
Lời giải 
Gọi A2B2C2 là tam giác tạo bởi 3 phân giác ngoài góc A, B, C. Dễ dàng có 
AA2⊥ B2C2 , BB2⊥ A2C2 , CC2⊥ A2B2 . 
Tứ giác BC2B2C nội tiếp nên A1C2 . A 
1B 
2 = A1B . A1C 
Tương tự B1C2 . B1A 
2 = B1A . B1C , C1B2 . C1A2 = C1A . C1B 
Suy ra A1,B1,C1 cùng nằm trên trục đẳng phương của đường tròn (O) ngoại tiếp tam giác 
ABC và đường tròn (J) ngoại tiếp tam giác A2B2C2. Mà (O) là đường tròn Ơ-le của tam
giác A2B2C2, AA2, BB2 ,CC2 giao nhau tại trực tâm I của tam giác A2B2C2 (cũng đồng 
thời là tâm đường tròn nội tiếp tam giác ABC) suy ra I, O, J thẳng hàng. 
Vậy đường thẳng qua A1, B1, C1 vuông góc với OI (đpcm) 
Bài tập 7 
Cho tam giác ABC nội tiếp (O). (I), (Ia) lần lượt là đường tròn nội tiếp và bàng tiếp góc 
A. Giả sử I Ia giao BC và (O) lần lượt tại A’, M. Gọi N là trung điểm cung MBA. NI, NIa 
giao (O) lần lượt tại S,T. CMR S, T, A’ thẳng hàng. 
Lời giải 
Trang 35 
Ta có ∠NTS= 
1 
2 
(sđ ΝΑ+ sđ S 
Α )= 
1 
2 
(sđΝΜ +sđ AS )=∠NIM 
⇒∠IaTS=∠IaIS 
Suy ra tứ giác IaTIS nội tiếp (w1) 
Mặt khác,∠ IBIa=∠ ICIa=90° nên tứ giác IBIaC nội tiếp (w2) 
Ta thấy I Ia là trục đẳng phương của (w1) và (w2), BC là trục đẳng phương của (O) và 
(w2), TS là trục đẳng phương của (O) và (w1) 
Theo định lý về tâm đẳng phương thì I Ia, TS, BC đồng quy tại A’. 
Vậy T, A’, S thẳng hàng (đpcm) 
Bài tập 8 (Định lý Brianchon) 
Cho lục giác ABCDEF ngoại tiếp (O). CMR AD, BE, CF đồng quy. 
Lời giải 
Gọi G, H, I, J, K, L lần lượt là tiếp điểm của AB, BC, CD, DE, EF, FA với (O). 
Trên tia KF,HB, GB, JD, ID, LF lần lượt lấy các điểm P, S, Q, R, N, M sao cho 
KP = SH = GQ = JR = IN = LM.
Dựng (O1) tiếp xúc với EF,CB tại P, S, (O2) tiếp xúc AF, CD tại M, N, (O3) tiếp xúc AB, 
ED tại Q, R. 
Ta có FP=PK-FK=LM-LF=FM, CS=SH+HC=IN+IC=CN 
Suy ra FC là trục đẳng phương của (O1) và (O2). 
Tương tự AD là trục đẳng phương của (O2) và (O3), BE là trục đẳng phương của (O3) và 
(O1). Áp dụng định lý về tâm đẳng phương ta có AD, BE, CF đồng quy (đpcm). 
Ta chứng minh ba đường thẳng đó là các trục đẳng phương của từng cặp hai đường tròn 
có tâm không thẳng hàng. 
Bài tập 9 
Cho tam giác ABC và đường thẳng d. Gọi A’, B’, C’ lần lượt là hình chiếu của A, B, C 
trên d. Gọi d 
1, d 
2, d 
3 theo thứ tự là các đường thẳng đi qua A’, B’, C’ và vuông góc với 
Trang 36 
BC, CA, AB. Chứng minh rằng các đường thẳng d 
1, d 
2, d 
3 đồng quy. 
Lời giải
Trang 37 
Gọi O 
1, O 
2, O 
3 theo thứ tự là trung điểm của BC, CA, AB, ta có O 
1B’= O 
1C’; O 
2C’= O 
2 
A’; O 
3A’= O 
3B’. Gọi Γ 
1, Γ 
2, Γ 
3 theo thứ tự là các đường tròn tâm O 
1 bán kính O 
1C’, tâm 
O 
2 bán kính O 
2A’, tâm O 
3 bán kính O 
3B’. Ta có d 
3 là trục đẳng phương của Γ 
1 và Γ 
2. 
Tương tự d 
2 là trục đẳng phương của Γ 
3 và Γ 
1; d 
1 là trục đẳng phương của Γ 
2 và Γ 
3. Vậy, 
d 
1, d 
2, d 
3 đồng quy. 
Bài tập 10 
Cho (O,R) và hai điểm P,Q cố định (P nằm ngoài (O), Q nằm trong (O)). Dây cung AB 
của (O) luôn đi qua Q. PA, PB lần lượt giao (O) lần thứ hai tại D,C. Chứng minh rằng CD 
luôn đi qua một điểm cố định. 
Lời giải 
Gọi E là giao điểm thứ hai khác P của PQ với đường tròn ngoại tiếp tam giác PAB. CD 
giao PQ tại F. 
Ta có OQ2 
- R2 
= QA . QB = QP . QE , mà P,Q cố định nên QP = const, suy ra QE = 
const, do đó E cố định. 
Mặt khác∠PDC=∠PBA=∠PEA nên tứ giác DAEF nội tiếp. 
Suy ra PO2 
- R2 
= PD . PA = PE . PF . Do P,E cố định nên PE =const, suy ra PF 
=const. Do đó F cố định. 
Vậy CD luôn đi qua điểm F cố định (đpcm) 
Bài tập 11 
Cho (O1,R1) tiếp xúc ngoài với (O2,R2) tại M (R2R1). Xét điểm A di động trên đường 
tròn sao cho A,O1,O2 không thẳng hàng. Từ A kẻ tiếp tuyến AB, AC tới (O 
1). Các đường 
thẳng MB, MC cắt tại (O 
2) tại E, F. D là giao điểm của EF với tiếp tuyến tại A của (O 
2). 
Chứng minh rằng D di động trên một đường thẳng cố định. 
Lời giải
Trang 38 
Qua M kẻ tiếp tuyến chungcủa (O 
1) và (O 
2). 
Ta có ∠MCA= ∠CMy=∠FMD=∠FAM 
Do đó, △FAM~△FCA (g.g) ⇒ FA2 
= FM . FC = FO 
2 
- R 
1 
2 
(1) 
1 
Tương tự, EA2 
= EO 
2 
- R 
1 
2 
(2) 
1 
Coi (A,0) là đường tròn tâm A, bán kính 0 thì từ (1),(2), ta được EF là trục đẳng phương 
của (A,0) với (O 
1). 
Mà D nằm tren EF nên DA2 
= DO 
2 
- R 
1 
2 
1 
⇒ P 
D/(O 
1)=P 
D/(O 
2) 
Vậy D nằm trên trục đẳng phương của hai đường tròn cố định (O 
1) và (O 
2). 
Bài tập 12 
Cho tam giác ABC có D là trung điểm của cạnh BC. Gọi d là đường thẳng qua D và 
vuông góc với đường thẳng AD. Trên đường thẳng d lấy một điểm M bất kì. Gọi E, F lần 
lượt là trung điểm của các đoạn thẳng MB, MC. Đường thẳng qua E vuông góc với d cắt 
đường thẳng AB tại P, đường thẳng qua F vuông góc với d cắt đường thẳng AC tại Q. 
Chứng minh rằng đường thẳng qua M, vuông góc với đường thẳng PQ luôn đi qua một 
điểm cố định khi M di động trên đường thẳng d. 
Lời giải 
d 
d' 
I Q 
P 
H' 
K' 
K 
H 
E F 
D 
A 
B 
C 
M
Gọi H, K lần lượt là hình chiếu của B, C lên đường thẳng d. 
Do D là trung điểm của BC nên DH = DK, suy ra AD là trung trực của HK⇒AH =AK. 
Gọi (ω) là đường tròn tâm A đi qua H và K. 
Gọi H’, K’ lần lượt là các điểm đối xứng với H, K qua các đường thẳng AB, AC 
⇒H’, K’ thuộc (ω) . 
Giả sử các đường thẳng HH’, KK’ cắt nhau tại I thì I là điểm cố định (*) 
Ta có : PE // BH (cùng vuông góc với d) mà PE đi qua trung điểm của MB nên cũng qua 
trung điểm của MH⇒PE là trung trực của MH ⇒ PH = PM. 
Gọi 1 (ω ) là đường tròn tâm P đi qua H và M, do tính đối xứng nên H’ cũng thuộc 1 (ω ) . 
Hoàn toàn tương tự, ta cũng có: QF là trung trực của MK; nếu gọi 2 (ω ) là đường tròn tâm 
Q đi qua K và M thì K’thuộc 2 (ω ) . Ta lại có: 
+(ω) , 1 (ω ) cắt nhau tai H, H’ nên HH’ là trục đẳng phương của (ω) , 1 (ω ) . 
+(ω) , 2 (ω ) cắt nhau tai K, K’ nên KK’ là trục đẳng phương của (ω) , 2 (ω ) . 
Mặt khác : M cùng thuộc 1 (ω ) , 2 (ω ) và P, Q lần lượt là tâm của 1 (ω ) , 2 (ω ) nên đường 
thẳng d’ qua M, vuông góc với PQ chính là trục đẳng phương của 1 (ω ) , 2 (ω ) . 
Từ đó suy ra: HH’, KK’, d’ đồng quy tại tâm đẳng phương của ba đường tròn 
(ω) , 1 (ω ) , 2 (ω ) (**) 
Từ (*) và (**) suy ra d’ đi qua I là điểm cố định. 
Vậy đường thẳng qua M, vuông góc với đường thẳng PQ luôn đi qua một điểm cố định 
khi M di động trên đường thẳng d. 
Trang 39
Trang 40 
TÀI LIỆU THAM KHẢO 
1. Đoàn Quỳnh, Văn Như Cương, Trần Nam Dũng, Nguyễn Minh Hà, ĐỗThanh Sơn, 
Lê Bá Khánh Trình: Tài liệu giáo khoa chuyên Toán – Hình học 10. 
2. Một số tài liệu trên Internet. 
3. Crux Mathematicorum with Mathematical Mayhem. 
4. Dusan Djukic, Vladimir Jankovic, Ivan Matic, Nikola Petrovic: The IMO 
Compedium. A collection of Problems Suggested for the International 
Mathematical Olympiads 1959-2004. 
5. Mathematical Reflection. 
6. Website: http://www.imo-official.org/ 
7. Website: http://www.artofproblemsolving.com/Forum/portal.php?ml=1

More Related Content

What's hot

Bo de hinh hoc thcs
Bo de hinh hoc thcsBo de hinh hoc thcs
Bo de hinh hoc thcs
khanh271295
 
Hệ Hoán Vị Vòng Quanh
Hệ Hoán Vị Vòng QuanhHệ Hoán Vị Vòng Quanh
Hệ Hoán Vị Vòng Quanh
Nhập Vân Long
 
BĐT Côsi ngược dấu
BĐT Côsi ngược dấuBĐT Côsi ngược dấu
BĐT Côsi ngược dấu
nhankhangvt
 
Câu hỏi trắc nghiệm HÓA ĐẠI CƯƠNG
Câu hỏi trắc nghiệm HÓA ĐẠI CƯƠNGCâu hỏi trắc nghiệm HÓA ĐẠI CƯƠNG
Câu hỏi trắc nghiệm HÓA ĐẠI CƯƠNG
Trần Đương
 
CÁC DẠNG TOÁN TRONG VECTƠ
CÁC DẠNG TOÁN TRONG VECTƠCÁC DẠNG TOÁN TRONG VECTƠ
CÁC DẠNG TOÁN TRONG VECTƠ
DANAMATH
 
Sáng kiến kinh nghiệm: Giải một bài toán quỹ tích như thế nào - Trường THCS V...
Sáng kiến kinh nghiệm: Giải một bài toán quỹ tích như thế nào - Trường THCS V...Sáng kiến kinh nghiệm: Giải một bài toán quỹ tích như thế nào - Trường THCS V...
Sáng kiến kinh nghiệm: Giải một bài toán quỹ tích như thế nào - Trường THCS V...
Học Tập Long An
 
Diophantine equations Phương trình diophant
Diophantine equations Phương trình diophantDiophantine equations Phương trình diophant
Diophantine equations Phương trình diophant
Bui Loi
 
200 Bài toán hình học tọa độ phẳng
200 Bài toán hình học tọa độ phẳng200 Bài toán hình học tọa độ phẳng
200 Bài toán hình học tọa độ phẳng
tuituhoc
 
LÝ THUYẾT VÀ 15 ĐỀ ÔN TẬP KIỂM TRA CHƯƠNG 1 HÌNH HỌC 8 CỰC HAY
LÝ THUYẾT VÀ 15 ĐỀ ÔN TẬP KIỂM TRA CHƯƠNG 1 HÌNH HỌC 8 CỰC HAYLÝ THUYẾT VÀ 15 ĐỀ ÔN TẬP KIỂM TRA CHƯƠNG 1 HÌNH HỌC 8 CỰC HAY
LÝ THUYẾT VÀ 15 ĐỀ ÔN TẬP KIỂM TRA CHƯƠNG 1 HÌNH HỌC 8 CỰC HAY
Hoàng Thái Việt
 
[Htq] toan 9 hsg tp hn dap an
[Htq] toan 9 hsg tp hn   dap an[Htq] toan 9 hsg tp hn   dap an
[Htq] toan 9 hsg tp hn dap an
Hồng Quang
 
Cđ một số ứng dụng định lí mê nê la uýt và xê va
Cđ một số ứng dụng định lí mê nê la uýt và xê vaCđ một số ứng dụng định lí mê nê la uýt và xê va
Cđ một số ứng dụng định lí mê nê la uýt và xê va
Cảnh
 
Cđ giải hpt không mẫu mực
Cđ giải hpt không mẫu mựcCđ giải hpt không mẫu mực
Cđ giải hpt không mẫu mực
Cảnh
 
Trắc nghiệm vô cơ đại cương
Trắc nghiệm vô cơ đại cươngTrắc nghiệm vô cơ đại cương
Trắc nghiệm vô cơ đại cương
Trần Đương
 
trắc nghiệm giải phẫu có đáp án
trắc nghiệm giải phẫu có đáp ántrắc nghiệm giải phẫu có đáp án
trắc nghiệm giải phẫu có đáp án
Thịnh Lê
 
Dgnl dhqg ha noi 2022 de so 6
Dgnl dhqg ha noi 2022 de so 6Dgnl dhqg ha noi 2022 de so 6
CHUYÊN ĐỀ HÌNH HỌC 8 CHƯƠNG 3 & ĐỀ KIỂM TRA 2018
CHUYÊN ĐỀ HÌNH HỌC 8 CHƯƠNG 3 & ĐỀ KIỂM TRA 2018CHUYÊN ĐỀ HÌNH HỌC 8 CHƯƠNG 3 & ĐỀ KIỂM TRA 2018
CHUYÊN ĐỀ HÌNH HỌC 8 CHƯƠNG 3 & ĐỀ KIỂM TRA 2018
Hoàng Thái Việt
 
Tuyển tập các bài Toán Hình học lớp 9 ôn thi vào 10
Tuyển tập các bài Toán Hình học lớp 9 ôn thi vào 10Tuyển tập các bài Toán Hình học lớp 9 ôn thi vào 10
Tuyển tập các bài Toán Hình học lớp 9 ôn thi vào 10
BOIDUONGTOAN.COM
 
TUYỂN TẬP ĐỀ THI VÀO CÁC TRƯỜNG CHUYÊN TP HÀ NỘI (Ams-NguyenHue-ChuVanAn)
TUYỂN TẬP ĐỀ THI VÀO CÁC TRƯỜNG CHUYÊN TP HÀ NỘI (Ams-NguyenHue-ChuVanAn)TUYỂN TẬP ĐỀ THI VÀO CÁC TRƯỜNG CHUYÊN TP HÀ NỘI (Ams-NguyenHue-ChuVanAn)
TUYỂN TẬP ĐỀ THI VÀO CÁC TRƯỜNG CHUYÊN TP HÀ NỘI (Ams-NguyenHue-ChuVanAn)
Nhật Hiếu
 
60 cau hoi on hoa dai cuong
60 cau hoi on hoa dai cuong60 cau hoi on hoa dai cuong
60 cau hoi on hoa dai cuong
Trần Đương
 

What's hot (20)

Bo de hinh hoc thcs
Bo de hinh hoc thcsBo de hinh hoc thcs
Bo de hinh hoc thcs
 
Hệ Hoán Vị Vòng Quanh
Hệ Hoán Vị Vòng QuanhHệ Hoán Vị Vòng Quanh
Hệ Hoán Vị Vòng Quanh
 
BĐT Côsi ngược dấu
BĐT Côsi ngược dấuBĐT Côsi ngược dấu
BĐT Côsi ngược dấu
 
Câu hỏi trắc nghiệm HÓA ĐẠI CƯƠNG
Câu hỏi trắc nghiệm HÓA ĐẠI CƯƠNGCâu hỏi trắc nghiệm HÓA ĐẠI CƯƠNG
Câu hỏi trắc nghiệm HÓA ĐẠI CƯƠNG
 
CÁC DẠNG TOÁN TRONG VECTƠ
CÁC DẠNG TOÁN TRONG VECTƠCÁC DẠNG TOÁN TRONG VECTƠ
CÁC DẠNG TOÁN TRONG VECTƠ
 
Sáng kiến kinh nghiệm: Giải một bài toán quỹ tích như thế nào - Trường THCS V...
Sáng kiến kinh nghiệm: Giải một bài toán quỹ tích như thế nào - Trường THCS V...Sáng kiến kinh nghiệm: Giải một bài toán quỹ tích như thế nào - Trường THCS V...
Sáng kiến kinh nghiệm: Giải một bài toán quỹ tích như thế nào - Trường THCS V...
 
Diophantine equations Phương trình diophant
Diophantine equations Phương trình diophantDiophantine equations Phương trình diophant
Diophantine equations Phương trình diophant
 
200 Bài toán hình học tọa độ phẳng
200 Bài toán hình học tọa độ phẳng200 Bài toán hình học tọa độ phẳng
200 Bài toán hình học tọa độ phẳng
 
LÝ THUYẾT VÀ 15 ĐỀ ÔN TẬP KIỂM TRA CHƯƠNG 1 HÌNH HỌC 8 CỰC HAY
LÝ THUYẾT VÀ 15 ĐỀ ÔN TẬP KIỂM TRA CHƯƠNG 1 HÌNH HỌC 8 CỰC HAYLÝ THUYẾT VÀ 15 ĐỀ ÔN TẬP KIỂM TRA CHƯƠNG 1 HÌNH HỌC 8 CỰC HAY
LÝ THUYẾT VÀ 15 ĐỀ ÔN TẬP KIỂM TRA CHƯƠNG 1 HÌNH HỌC 8 CỰC HAY
 
Tai lieu-on-thi-lop-10-mon-toan
Tai lieu-on-thi-lop-10-mon-toanTai lieu-on-thi-lop-10-mon-toan
Tai lieu-on-thi-lop-10-mon-toan
 
[Htq] toan 9 hsg tp hn dap an
[Htq] toan 9 hsg tp hn   dap an[Htq] toan 9 hsg tp hn   dap an
[Htq] toan 9 hsg tp hn dap an
 
Cđ một số ứng dụng định lí mê nê la uýt và xê va
Cđ một số ứng dụng định lí mê nê la uýt và xê vaCđ một số ứng dụng định lí mê nê la uýt và xê va
Cđ một số ứng dụng định lí mê nê la uýt và xê va
 
Cđ giải hpt không mẫu mực
Cđ giải hpt không mẫu mựcCđ giải hpt không mẫu mực
Cđ giải hpt không mẫu mực
 
Trắc nghiệm vô cơ đại cương
Trắc nghiệm vô cơ đại cươngTrắc nghiệm vô cơ đại cương
Trắc nghiệm vô cơ đại cương
 
trắc nghiệm giải phẫu có đáp án
trắc nghiệm giải phẫu có đáp ántrắc nghiệm giải phẫu có đáp án
trắc nghiệm giải phẫu có đáp án
 
Dgnl dhqg ha noi 2022 de so 6
Dgnl dhqg ha noi 2022 de so 6Dgnl dhqg ha noi 2022 de so 6
Dgnl dhqg ha noi 2022 de so 6
 
CHUYÊN ĐỀ HÌNH HỌC 8 CHƯƠNG 3 & ĐỀ KIỂM TRA 2018
CHUYÊN ĐỀ HÌNH HỌC 8 CHƯƠNG 3 & ĐỀ KIỂM TRA 2018CHUYÊN ĐỀ HÌNH HỌC 8 CHƯƠNG 3 & ĐỀ KIỂM TRA 2018
CHUYÊN ĐỀ HÌNH HỌC 8 CHƯƠNG 3 & ĐỀ KIỂM TRA 2018
 
Tuyển tập các bài Toán Hình học lớp 9 ôn thi vào 10
Tuyển tập các bài Toán Hình học lớp 9 ôn thi vào 10Tuyển tập các bài Toán Hình học lớp 9 ôn thi vào 10
Tuyển tập các bài Toán Hình học lớp 9 ôn thi vào 10
 
TUYỂN TẬP ĐỀ THI VÀO CÁC TRƯỜNG CHUYÊN TP HÀ NỘI (Ams-NguyenHue-ChuVanAn)
TUYỂN TẬP ĐỀ THI VÀO CÁC TRƯỜNG CHUYÊN TP HÀ NỘI (Ams-NguyenHue-ChuVanAn)TUYỂN TẬP ĐỀ THI VÀO CÁC TRƯỜNG CHUYÊN TP HÀ NỘI (Ams-NguyenHue-ChuVanAn)
TUYỂN TẬP ĐỀ THI VÀO CÁC TRƯỜNG CHUYÊN TP HÀ NỘI (Ams-NguyenHue-ChuVanAn)
 
60 cau hoi on hoa dai cuong
60 cau hoi on hoa dai cuong60 cau hoi on hoa dai cuong
60 cau hoi on hoa dai cuong
 

Similar to Chuyên đề phương tích và ứng dụng

Vi tri tuong doi cua hai duong tron
Vi tri tuong doi cua hai duong tronVi tri tuong doi cua hai duong tron
Vi tri tuong doi cua hai duong tron
hoaadc2016
 
Thay khanh iwaz
Thay khanh iwazThay khanh iwaz
Thay khanh iwaz
ChnhTrung3
 
Chuyen de hinh hoc 9
Chuyen de hinh hoc 9Chuyen de hinh hoc 9
Chuyen de hinh hoc 9
Nguyễn Hoàng Phước
 
[Vietmaths]duong thang duongtron-oxy vietmaths
[Vietmaths]duong thang duongtron-oxy vietmaths[Vietmaths]duong thang duongtron-oxy vietmaths
[Vietmaths]duong thang duongtron-oxy vietmaths
anhyeuem2509
 
Đường thẳng đường tròn Oxy Mathvn
Đường thẳng đường tròn Oxy MathvnĐường thẳng đường tròn Oxy Mathvn
Đường thẳng đường tròn Oxy MathvnMinh Thắng Trần
 
692 bai hinh toa do trong mp toa do khong gian
692 bai hinh  toa do trong mp  toa do khong gian 692 bai hinh  toa do trong mp  toa do khong gian
692 bai hinh toa do trong mp toa do khong gian Nguyễn Đình Tân
 
Chuyên đề về đường tròn
Chuyên đề về đường trònChuyên đề về đường tròn
Chuyên đề về đường tròn
Ngo Quang Viet
 
các bài toán hình học lớp 9 có lời giải
các bài toán hình học lớp 9 có lời giảicác bài toán hình học lớp 9 có lời giải
các bài toán hình học lớp 9 có lời giải
Khoảnh Khắc Bình Yên
 
9 tu giac noi tiep
9 tu giac noi tiep9 tu giac noi tiep
9 tu giac noi tiep
Hồng Quang
 
36 de-luyen-thi-vao-lop 10
36 de-luyen-thi-vao-lop 1036 de-luyen-thi-vao-lop 10
36 de-luyen-thi-vao-lop 10
mcbooksjsc
 
CHUYÊN ĐỀ :TỌA ĐỘ PHẲNG - PHƯƠNG PHÁP VECTƠ
CHUYÊN ĐỀ :TỌA ĐỘ PHẲNG - PHƯƠNG PHÁP VECTƠCHUYÊN ĐỀ :TỌA ĐỘ PHẲNG - PHƯƠNG PHÁP VECTƠ
CHUYÊN ĐỀ :TỌA ĐỘ PHẲNG - PHƯƠNG PHÁP VECTƠ
DANAMATH
 
Cực trị hình học cực hay nè,,,
Cực trị hình học cực hay nè,,,Cực trị hình học cực hay nè,,,
Cực trị hình học cực hay nè,,,
bichshiho20
 
Dap an-de-thi-toan-vao-10-2013-ha-noi
Dap an-de-thi-toan-vao-10-2013-ha-noiDap an-de-thi-toan-vao-10-2013-ha-noi
Dap an-de-thi-toan-vao-10-2013-ha-noiwebdethi
 
Hinh chuong3
Hinh chuong3Hinh chuong3
Hinh chuong3
Nguyen Van Tai
 
Chuyên đề 3 phương pháp toạ độ trong mặt phẳng
Chuyên đề 3 phương pháp toạ độ trong mặt phẳngChuyên đề 3 phương pháp toạ độ trong mặt phẳng
Chuyên đề 3 phương pháp toạ độ trong mặt phẳng
phamchidac
 
Hinh chuong3
Hinh chuong3Hinh chuong3
Hinh chuong3
Hai Trung Pham
 
Chuyên đề 3 phương pháp toạ độ trong mặt phẳng
Chuyên đề 3 phương pháp toạ độ trong mặt phẳngChuyên đề 3 phương pháp toạ độ trong mặt phẳng
Chuyên đề 3 phương pháp toạ độ trong mặt phẳng
phamchidac
 
Bai 5,6 hai hinh bang nhau và phep vi tu
Bai 5,6 hai hinh bang nhau và phep vi tuBai 5,6 hai hinh bang nhau và phep vi tu
Bai 5,6 hai hinh bang nhau và phep vi tuLe Hanh
 
Ôn Thi Đại Học 2015 (Tọa độ mặt phẳng)
Ôn Thi Đại Học 2015 (Tọa độ mặt phẳng)Ôn Thi Đại Học 2015 (Tọa độ mặt phẳng)
Ôn Thi Đại Học 2015 (Tọa độ mặt phẳng)
Phạm Lộc
 
Đề thi thử Toán - Chuyên Vĩnh Phúc 2014 lần 4 Khối B,D
Đề thi thử Toán - Chuyên Vĩnh Phúc 2014 lần 4 Khối B,DĐề thi thử Toán - Chuyên Vĩnh Phúc 2014 lần 4 Khối B,D
Đề thi thử Toán - Chuyên Vĩnh Phúc 2014 lần 4 Khối B,D
dlinh123
 

Similar to Chuyên đề phương tích và ứng dụng (20)

Vi tri tuong doi cua hai duong tron
Vi tri tuong doi cua hai duong tronVi tri tuong doi cua hai duong tron
Vi tri tuong doi cua hai duong tron
 
Thay khanh iwaz
Thay khanh iwazThay khanh iwaz
Thay khanh iwaz
 
Chuyen de hinh hoc 9
Chuyen de hinh hoc 9Chuyen de hinh hoc 9
Chuyen de hinh hoc 9
 
[Vietmaths]duong thang duongtron-oxy vietmaths
[Vietmaths]duong thang duongtron-oxy vietmaths[Vietmaths]duong thang duongtron-oxy vietmaths
[Vietmaths]duong thang duongtron-oxy vietmaths
 
Đường thẳng đường tròn Oxy Mathvn
Đường thẳng đường tròn Oxy MathvnĐường thẳng đường tròn Oxy Mathvn
Đường thẳng đường tròn Oxy Mathvn
 
692 bai hinh toa do trong mp toa do khong gian
692 bai hinh  toa do trong mp  toa do khong gian 692 bai hinh  toa do trong mp  toa do khong gian
692 bai hinh toa do trong mp toa do khong gian
 
Chuyên đề về đường tròn
Chuyên đề về đường trònChuyên đề về đường tròn
Chuyên đề về đường tròn
 
các bài toán hình học lớp 9 có lời giải
các bài toán hình học lớp 9 có lời giảicác bài toán hình học lớp 9 có lời giải
các bài toán hình học lớp 9 có lời giải
 
9 tu giac noi tiep
9 tu giac noi tiep9 tu giac noi tiep
9 tu giac noi tiep
 
36 de-luyen-thi-vao-lop 10
36 de-luyen-thi-vao-lop 1036 de-luyen-thi-vao-lop 10
36 de-luyen-thi-vao-lop 10
 
CHUYÊN ĐỀ :TỌA ĐỘ PHẲNG - PHƯƠNG PHÁP VECTƠ
CHUYÊN ĐỀ :TỌA ĐỘ PHẲNG - PHƯƠNG PHÁP VECTƠCHUYÊN ĐỀ :TỌA ĐỘ PHẲNG - PHƯƠNG PHÁP VECTƠ
CHUYÊN ĐỀ :TỌA ĐỘ PHẲNG - PHƯƠNG PHÁP VECTƠ
 
Cực trị hình học cực hay nè,,,
Cực trị hình học cực hay nè,,,Cực trị hình học cực hay nè,,,
Cực trị hình học cực hay nè,,,
 
Dap an-de-thi-toan-vao-10-2013-ha-noi
Dap an-de-thi-toan-vao-10-2013-ha-noiDap an-de-thi-toan-vao-10-2013-ha-noi
Dap an-de-thi-toan-vao-10-2013-ha-noi
 
Hinh chuong3
Hinh chuong3Hinh chuong3
Hinh chuong3
 
Chuyên đề 3 phương pháp toạ độ trong mặt phẳng
Chuyên đề 3 phương pháp toạ độ trong mặt phẳngChuyên đề 3 phương pháp toạ độ trong mặt phẳng
Chuyên đề 3 phương pháp toạ độ trong mặt phẳng
 
Hinh chuong3
Hinh chuong3Hinh chuong3
Hinh chuong3
 
Chuyên đề 3 phương pháp toạ độ trong mặt phẳng
Chuyên đề 3 phương pháp toạ độ trong mặt phẳngChuyên đề 3 phương pháp toạ độ trong mặt phẳng
Chuyên đề 3 phương pháp toạ độ trong mặt phẳng
 
Bai 5,6 hai hinh bang nhau và phep vi tu
Bai 5,6 hai hinh bang nhau và phep vi tuBai 5,6 hai hinh bang nhau và phep vi tu
Bai 5,6 hai hinh bang nhau và phep vi tu
 
Ôn Thi Đại Học 2015 (Tọa độ mặt phẳng)
Ôn Thi Đại Học 2015 (Tọa độ mặt phẳng)Ôn Thi Đại Học 2015 (Tọa độ mặt phẳng)
Ôn Thi Đại Học 2015 (Tọa độ mặt phẳng)
 
Đề thi thử Toán - Chuyên Vĩnh Phúc 2014 lần 4 Khối B,D
Đề thi thử Toán - Chuyên Vĩnh Phúc 2014 lần 4 Khối B,DĐề thi thử Toán - Chuyên Vĩnh Phúc 2014 lần 4 Khối B,D
Đề thi thử Toán - Chuyên Vĩnh Phúc 2014 lần 4 Khối B,D
 

More from Vui Lên Bạn Nhé

3 Đề thi thử môn toán 2015 from http://toanphothong.com/
3 Đề thi thử môn toán 2015 from http://toanphothong.com/3 Đề thi thử môn toán 2015 from http://toanphothong.com/
3 Đề thi thử môn toán 2015 from http://toanphothong.com/
Vui Lên Bạn Nhé
 
3 Đề thi thử môn toán 2015 from http://toanphothong.com/
3 Đề thi thử môn toán 2015 from http://toanphothong.com/3 Đề thi thử môn toán 2015 from http://toanphothong.com/
3 Đề thi thử môn toán 2015 from http://toanphothong.com/
Vui Lên Bạn Nhé
 
Đề Toán 2015 Yên Lạc - VP lần 1
Đề Toán 2015 Yên Lạc - VP lần 1Đề Toán 2015 Yên Lạc - VP lần 1
Đề Toán 2015 Yên Lạc - VP lần 1
Vui Lên Bạn Nhé
 
Đề số 5 toán 2015 LTĐH Đà Nẵng
Đề số 5 toán 2015 LTĐH Đà NẵngĐề số 5 toán 2015 LTĐH Đà Nẵng
Đề số 5 toán 2015 LTĐH Đà Nẵng
Vui Lên Bạn Nhé
 
Đề số 1 2015 của Vũ Văn Bắc
Đề số 1 2015 của Vũ Văn BắcĐề số 1 2015 của Vũ Văn Bắc
Đề số 1 2015 của Vũ Văn Bắc
Vui Lên Bạn Nhé
 
Đề số 12 Thầy Phạm Tuấn Khải
Đề số 12 Thầy Phạm Tuấn KhảiĐề số 12 Thầy Phạm Tuấn Khải
Đề số 12 Thầy Phạm Tuấn Khải
Vui Lên Bạn Nhé
 
Giao trinh guitar
Giao trinh guitarGiao trinh guitar
Giao trinh guitar
Vui Lên Bạn Nhé
 
Giao trinh hoc guitar
Giao trinh hoc guitarGiao trinh hoc guitar
Giao trinh hoc guitar
Vui Lên Bạn Nhé
 
Lý thuyết phân dạng BT hóa 11 kỳ 2
Lý thuyết phân dạng BT hóa 11 kỳ 2Lý thuyết phân dạng BT hóa 11 kỳ 2
Lý thuyết phân dạng BT hóa 11 kỳ 2
Vui Lên Bạn Nhé
 
3 Đề thi thử 2015 + đáp án
3 Đề thi thử 2015 + đáp án3 Đề thi thử 2015 + đáp án
3 Đề thi thử 2015 + đáp án
Vui Lên Bạn Nhé
 
ĐỀ THI THỬ MÔN TOÁN 2015 CỦA ĐẠI HỌC QUỐC GIA HÀ NỘI
ĐỀ THI THỬ MÔN TOÁN 2015 CỦA ĐẠI HỌC QUỐC GIA HÀ NỘIĐỀ THI THỬ MÔN TOÁN 2015 CỦA ĐẠI HỌC QUỐC GIA HÀ NỘI
ĐỀ THI THỬ MÔN TOÁN 2015 CỦA ĐẠI HỌC QUỐC GIA HÀ NỘI
Vui Lên Bạn Nhé
 
Phuong phap chuyen vi chung minh bdt
Phuong phap chuyen vi chung minh bdtPhuong phap chuyen vi chung minh bdt
Phuong phap chuyen vi chung minh bdt
Vui Lên Bạn Nhé
 
3 đề thi thử toán 2015 + đáp án (Bình Thuận)
3 đề thi thử toán 2015 + đáp án (Bình Thuận)3 đề thi thử toán 2015 + đáp án (Bình Thuận)
3 đề thi thử toán 2015 + đáp án (Bình Thuận)
Vui Lên Bạn Nhé
 
Đề thi thử ĐH môn toán lần 1_2015 trường chuyên TB
Đề thi thử ĐH môn toán lần 1_2015 trường chuyên TBĐề thi thử ĐH môn toán lần 1_2015 trường chuyên TB
Đề thi thử ĐH môn toán lần 1_2015 trường chuyên TB
Vui Lên Bạn Nhé
 
Mot so bai tap hoa moi va hay mua thi thu 2011
Mot so bai tap hoa moi va hay mua thi thu 2011 Mot so bai tap hoa moi va hay mua thi thu 2011
Mot so bai tap hoa moi va hay mua thi thu 2011
Vui Lên Bạn Nhé
 
Algebraic techniques in combinatorics
Algebraic techniques in combinatoricsAlgebraic techniques in combinatorics
Algebraic techniques in combinatorics
Vui Lên Bạn Nhé
 
Algebraic inequalities old and new methods
Algebraic inequalities old and new methodsAlgebraic inequalities old and new methods
Algebraic inequalities old and new methods
Vui Lên Bạn Nhé
 
ăn chay: Rau cải rổ xào
ăn chay: Rau cải rổ xàoăn chay: Rau cải rổ xào
ăn chay: Rau cải rổ xào
Vui Lên Bạn Nhé
 
Một số món chay 1
Một số món chay 1Một số món chay 1
Một số món chay 1
Vui Lên Bạn Nhé
 
ăn chay: Miến xào giò chay
ăn chay: Miến xào giò chayăn chay: Miến xào giò chay
ăn chay: Miến xào giò chay
Vui Lên Bạn Nhé
 

More from Vui Lên Bạn Nhé (20)

3 Đề thi thử môn toán 2015 from http://toanphothong.com/
3 Đề thi thử môn toán 2015 from http://toanphothong.com/3 Đề thi thử môn toán 2015 from http://toanphothong.com/
3 Đề thi thử môn toán 2015 from http://toanphothong.com/
 
3 Đề thi thử môn toán 2015 from http://toanphothong.com/
3 Đề thi thử môn toán 2015 from http://toanphothong.com/3 Đề thi thử môn toán 2015 from http://toanphothong.com/
3 Đề thi thử môn toán 2015 from http://toanphothong.com/
 
Đề Toán 2015 Yên Lạc - VP lần 1
Đề Toán 2015 Yên Lạc - VP lần 1Đề Toán 2015 Yên Lạc - VP lần 1
Đề Toán 2015 Yên Lạc - VP lần 1
 
Đề số 5 toán 2015 LTĐH Đà Nẵng
Đề số 5 toán 2015 LTĐH Đà NẵngĐề số 5 toán 2015 LTĐH Đà Nẵng
Đề số 5 toán 2015 LTĐH Đà Nẵng
 
Đề số 1 2015 của Vũ Văn Bắc
Đề số 1 2015 của Vũ Văn BắcĐề số 1 2015 của Vũ Văn Bắc
Đề số 1 2015 của Vũ Văn Bắc
 
Đề số 12 Thầy Phạm Tuấn Khải
Đề số 12 Thầy Phạm Tuấn KhảiĐề số 12 Thầy Phạm Tuấn Khải
Đề số 12 Thầy Phạm Tuấn Khải
 
Giao trinh guitar
Giao trinh guitarGiao trinh guitar
Giao trinh guitar
 
Giao trinh hoc guitar
Giao trinh hoc guitarGiao trinh hoc guitar
Giao trinh hoc guitar
 
Lý thuyết phân dạng BT hóa 11 kỳ 2
Lý thuyết phân dạng BT hóa 11 kỳ 2Lý thuyết phân dạng BT hóa 11 kỳ 2
Lý thuyết phân dạng BT hóa 11 kỳ 2
 
3 Đề thi thử 2015 + đáp án
3 Đề thi thử 2015 + đáp án3 Đề thi thử 2015 + đáp án
3 Đề thi thử 2015 + đáp án
 
ĐỀ THI THỬ MÔN TOÁN 2015 CỦA ĐẠI HỌC QUỐC GIA HÀ NỘI
ĐỀ THI THỬ MÔN TOÁN 2015 CỦA ĐẠI HỌC QUỐC GIA HÀ NỘIĐỀ THI THỬ MÔN TOÁN 2015 CỦA ĐẠI HỌC QUỐC GIA HÀ NỘI
ĐỀ THI THỬ MÔN TOÁN 2015 CỦA ĐẠI HỌC QUỐC GIA HÀ NỘI
 
Phuong phap chuyen vi chung minh bdt
Phuong phap chuyen vi chung minh bdtPhuong phap chuyen vi chung minh bdt
Phuong phap chuyen vi chung minh bdt
 
3 đề thi thử toán 2015 + đáp án (Bình Thuận)
3 đề thi thử toán 2015 + đáp án (Bình Thuận)3 đề thi thử toán 2015 + đáp án (Bình Thuận)
3 đề thi thử toán 2015 + đáp án (Bình Thuận)
 
Đề thi thử ĐH môn toán lần 1_2015 trường chuyên TB
Đề thi thử ĐH môn toán lần 1_2015 trường chuyên TBĐề thi thử ĐH môn toán lần 1_2015 trường chuyên TB
Đề thi thử ĐH môn toán lần 1_2015 trường chuyên TB
 
Mot so bai tap hoa moi va hay mua thi thu 2011
Mot so bai tap hoa moi va hay mua thi thu 2011 Mot so bai tap hoa moi va hay mua thi thu 2011
Mot so bai tap hoa moi va hay mua thi thu 2011
 
Algebraic techniques in combinatorics
Algebraic techniques in combinatoricsAlgebraic techniques in combinatorics
Algebraic techniques in combinatorics
 
Algebraic inequalities old and new methods
Algebraic inequalities old and new methodsAlgebraic inequalities old and new methods
Algebraic inequalities old and new methods
 
ăn chay: Rau cải rổ xào
ăn chay: Rau cải rổ xàoăn chay: Rau cải rổ xào
ăn chay: Rau cải rổ xào
 
Một số món chay 1
Một số món chay 1Một số món chay 1
Một số món chay 1
 
ăn chay: Miến xào giò chay
ăn chay: Miến xào giò chayăn chay: Miến xào giò chay
ăn chay: Miến xào giò chay
 

Recently uploaded

CHUYÊN ĐỀ DẠY THÊM HÓA HỌC LỚP 10 - SÁCH MỚI - FORM BÀI TẬP 2025 (DÙNG CHUNG ...
CHUYÊN ĐỀ DẠY THÊM HÓA HỌC LỚP 10 - SÁCH MỚI - FORM BÀI TẬP 2025 (DÙNG CHUNG ...CHUYÊN ĐỀ DẠY THÊM HÓA HỌC LỚP 10 - SÁCH MỚI - FORM BÀI TẬP 2025 (DÙNG CHUNG ...
CHUYÊN ĐỀ DẠY THÊM HÓA HỌC LỚP 10 - SÁCH MỚI - FORM BÀI TẬP 2025 (DÙNG CHUNG ...
Nguyen Thanh Tu Collection
 
insulin cho benh nhan nam vien co tang duong huyet
insulin cho benh nhan nam vien co tang duong huyetinsulin cho benh nhan nam vien co tang duong huyet
insulin cho benh nhan nam vien co tang duong huyet
lmhong80
 
FSSC 22000 version 6_Seminar_FINAL end.pptx
FSSC 22000 version 6_Seminar_FINAL end.pptxFSSC 22000 version 6_Seminar_FINAL end.pptx
FSSC 22000 version 6_Seminar_FINAL end.pptx
deviv80273
 
Halloween vocabulary for kids in primary school
Halloween vocabulary for kids in primary schoolHalloween vocabulary for kids in primary school
Halloween vocabulary for kids in primary school
AnhPhm265031
 
BÀI TẬP BỔ TRỢ TIẾNG ANH I-LEARN SMART WORLD 9 CẢ NĂM CÓ TEST THEO UNIT NĂM H...
BÀI TẬP BỔ TRỢ TIẾNG ANH I-LEARN SMART WORLD 9 CẢ NĂM CÓ TEST THEO UNIT NĂM H...BÀI TẬP BỔ TRỢ TIẾNG ANH I-LEARN SMART WORLD 9 CẢ NĂM CÓ TEST THEO UNIT NĂM H...
BÀI TẬP BỔ TRỢ TIẾNG ANH I-LEARN SMART WORLD 9 CẢ NĂM CÓ TEST THEO UNIT NĂM H...
Nguyen Thanh Tu Collection
 
SLIDE BÀI GIẢNG MÔN THƯƠNG MẠI ĐIỆN TỬ.pdf
SLIDE BÀI GIẢNG MÔN THƯƠNG MẠI ĐIỆN TỬ.pdfSLIDE BÀI GIẢNG MÔN THƯƠNG MẠI ĐIỆN TỬ.pdf
SLIDE BÀI GIẢNG MÔN THƯƠNG MẠI ĐIỆN TỬ.pdf
UyenDang34
 
40 câu hỏi - đáp Bộ luật dân sự năm 2015 (1).doc
40 câu hỏi - đáp Bộ  luật dân sự năm  2015 (1).doc40 câu hỏi - đáp Bộ  luật dân sự năm  2015 (1).doc
40 câu hỏi - đáp Bộ luật dân sự năm 2015 (1).doc
NguynDimQunh33
 
THONG BAO nop ho so xet tuyen TS6 24-25.pdf
THONG BAO nop ho so xet tuyen TS6 24-25.pdfTHONG BAO nop ho so xet tuyen TS6 24-25.pdf
THONG BAO nop ho so xet tuyen TS6 24-25.pdf
QucHHunhnh
 
Giải phẫu tim sau đại học- LÊ QUANG TUYỀN
Giải phẫu tim sau đại học- LÊ QUANG TUYỀNGiải phẫu tim sau đại học- LÊ QUANG TUYỀN
Giải phẫu tim sau đại học- LÊ QUANG TUYỀN
linh miu
 
100 DẪN CHỨNG NGHỊ LUẬN XÃ HỘiI HAY.docx
100 DẪN CHỨNG NGHỊ LUẬN XÃ HỘiI HAY.docx100 DẪN CHỨNG NGHỊ LUẬN XÃ HỘiI HAY.docx
100 DẪN CHỨNG NGHỊ LUẬN XÃ HỘiI HAY.docx
khanhthy3000
 
LỊCH SỬ 12 - CHUYÊN ĐỀ 10 - TRẮC NGHIỆM.pptx
LỊCH SỬ 12 - CHUYÊN ĐỀ 10 - TRẮC NGHIỆM.pptxLỊCH SỬ 12 - CHUYÊN ĐỀ 10 - TRẮC NGHIỆM.pptx
LỊCH SỬ 12 - CHUYÊN ĐỀ 10 - TRẮC NGHIỆM.pptx
12D241NguynPhmMaiTra
 
Biểu tượng trăng và bầu trời trong tác phẩm của Nguyễn Quang Thiều
Biểu tượng trăng và bầu trời trong tác phẩm của Nguyễn Quang ThiềuBiểu tượng trăng và bầu trời trong tác phẩm của Nguyễn Quang Thiều
Biểu tượng trăng và bầu trời trong tác phẩm của Nguyễn Quang Thiều
lamluanvan.net Viết thuê luận văn
 
Smartbiz_He thong MES nganh may mac_2024june
Smartbiz_He thong MES nganh may mac_2024juneSmartbiz_He thong MES nganh may mac_2024june
Smartbiz_He thong MES nganh may mac_2024june
SmartBiz
 
PLĐC-chương 1 (1).ppt của trường ĐH Ngoại thương
PLĐC-chương 1 (1).ppt của trường  ĐH Ngoại thươngPLĐC-chương 1 (1).ppt của trường  ĐH Ngoại thương
PLĐC-chương 1 (1).ppt của trường ĐH Ngoại thương
hieutrinhvan27052005
 
Cau-Trắc-Nghiệm-TTHCM-Tham-Khảo-THI-CUỐI-KI.pdf
Cau-Trắc-Nghiệm-TTHCM-Tham-Khảo-THI-CUỐI-KI.pdfCau-Trắc-Nghiệm-TTHCM-Tham-Khảo-THI-CUỐI-KI.pdf
Cau-Trắc-Nghiệm-TTHCM-Tham-Khảo-THI-CUỐI-KI.pdf
HngMLTh
 
Văn 7. Truyện ngụ ngôn Rùa và thỏ+ Viết PT nhân vật.docx
Văn 7. Truyện ngụ ngôn Rùa và thỏ+ Viết PT nhân vật.docxVăn 7. Truyện ngụ ngôn Rùa và thỏ+ Viết PT nhân vật.docx
Văn 7. Truyện ngụ ngôn Rùa và thỏ+ Viết PT nhân vật.docx
metamngoc123
 
BÁO CÁO CUỐI KỲ PHÂN TÍCH THIẾT KẾ HƯỚNG ĐỐI TƯỢNG - NHÓM 7.docx
BÁO CÁO CUỐI KỲ PHÂN TÍCH THIẾT KẾ HƯỚNG ĐỐI TƯỢNG - NHÓM 7.docxBÁO CÁO CUỐI KỲ PHÂN TÍCH THIẾT KẾ HƯỚNG ĐỐI TƯỢNG - NHÓM 7.docx
BÁO CÁO CUỐI KỲ PHÂN TÍCH THIẾT KẾ HƯỚNG ĐỐI TƯỢNG - NHÓM 7.docx
HngL891608
 
trắc nhiệm ký sinh.docxddddddddddddddddd
trắc nhiệm ký sinh.docxdddddddddddddddddtrắc nhiệm ký sinh.docxddddddddddddddddd
trắc nhiệm ký sinh.docxddddddddddddddddd
my21xn0084
 

Recently uploaded (18)

CHUYÊN ĐỀ DẠY THÊM HÓA HỌC LỚP 10 - SÁCH MỚI - FORM BÀI TẬP 2025 (DÙNG CHUNG ...
CHUYÊN ĐỀ DẠY THÊM HÓA HỌC LỚP 10 - SÁCH MỚI - FORM BÀI TẬP 2025 (DÙNG CHUNG ...CHUYÊN ĐỀ DẠY THÊM HÓA HỌC LỚP 10 - SÁCH MỚI - FORM BÀI TẬP 2025 (DÙNG CHUNG ...
CHUYÊN ĐỀ DẠY THÊM HÓA HỌC LỚP 10 - SÁCH MỚI - FORM BÀI TẬP 2025 (DÙNG CHUNG ...
 
insulin cho benh nhan nam vien co tang duong huyet
insulin cho benh nhan nam vien co tang duong huyetinsulin cho benh nhan nam vien co tang duong huyet
insulin cho benh nhan nam vien co tang duong huyet
 
FSSC 22000 version 6_Seminar_FINAL end.pptx
FSSC 22000 version 6_Seminar_FINAL end.pptxFSSC 22000 version 6_Seminar_FINAL end.pptx
FSSC 22000 version 6_Seminar_FINAL end.pptx
 
Halloween vocabulary for kids in primary school
Halloween vocabulary for kids in primary schoolHalloween vocabulary for kids in primary school
Halloween vocabulary for kids in primary school
 
BÀI TẬP BỔ TRỢ TIẾNG ANH I-LEARN SMART WORLD 9 CẢ NĂM CÓ TEST THEO UNIT NĂM H...
BÀI TẬP BỔ TRỢ TIẾNG ANH I-LEARN SMART WORLD 9 CẢ NĂM CÓ TEST THEO UNIT NĂM H...BÀI TẬP BỔ TRỢ TIẾNG ANH I-LEARN SMART WORLD 9 CẢ NĂM CÓ TEST THEO UNIT NĂM H...
BÀI TẬP BỔ TRỢ TIẾNG ANH I-LEARN SMART WORLD 9 CẢ NĂM CÓ TEST THEO UNIT NĂM H...
 
SLIDE BÀI GIẢNG MÔN THƯƠNG MẠI ĐIỆN TỬ.pdf
SLIDE BÀI GIẢNG MÔN THƯƠNG MẠI ĐIỆN TỬ.pdfSLIDE BÀI GIẢNG MÔN THƯƠNG MẠI ĐIỆN TỬ.pdf
SLIDE BÀI GIẢNG MÔN THƯƠNG MẠI ĐIỆN TỬ.pdf
 
40 câu hỏi - đáp Bộ luật dân sự năm 2015 (1).doc
40 câu hỏi - đáp Bộ  luật dân sự năm  2015 (1).doc40 câu hỏi - đáp Bộ  luật dân sự năm  2015 (1).doc
40 câu hỏi - đáp Bộ luật dân sự năm 2015 (1).doc
 
THONG BAO nop ho so xet tuyen TS6 24-25.pdf
THONG BAO nop ho so xet tuyen TS6 24-25.pdfTHONG BAO nop ho so xet tuyen TS6 24-25.pdf
THONG BAO nop ho so xet tuyen TS6 24-25.pdf
 
Giải phẫu tim sau đại học- LÊ QUANG TUYỀN
Giải phẫu tim sau đại học- LÊ QUANG TUYỀNGiải phẫu tim sau đại học- LÊ QUANG TUYỀN
Giải phẫu tim sau đại học- LÊ QUANG TUYỀN
 
100 DẪN CHỨNG NGHỊ LUẬN XÃ HỘiI HAY.docx
100 DẪN CHỨNG NGHỊ LUẬN XÃ HỘiI HAY.docx100 DẪN CHỨNG NGHỊ LUẬN XÃ HỘiI HAY.docx
100 DẪN CHỨNG NGHỊ LUẬN XÃ HỘiI HAY.docx
 
LỊCH SỬ 12 - CHUYÊN ĐỀ 10 - TRẮC NGHIỆM.pptx
LỊCH SỬ 12 - CHUYÊN ĐỀ 10 - TRẮC NGHIỆM.pptxLỊCH SỬ 12 - CHUYÊN ĐỀ 10 - TRẮC NGHIỆM.pptx
LỊCH SỬ 12 - CHUYÊN ĐỀ 10 - TRẮC NGHIỆM.pptx
 
Biểu tượng trăng và bầu trời trong tác phẩm của Nguyễn Quang Thiều
Biểu tượng trăng và bầu trời trong tác phẩm của Nguyễn Quang ThiềuBiểu tượng trăng và bầu trời trong tác phẩm của Nguyễn Quang Thiều
Biểu tượng trăng và bầu trời trong tác phẩm của Nguyễn Quang Thiều
 
Smartbiz_He thong MES nganh may mac_2024june
Smartbiz_He thong MES nganh may mac_2024juneSmartbiz_He thong MES nganh may mac_2024june
Smartbiz_He thong MES nganh may mac_2024june
 
PLĐC-chương 1 (1).ppt của trường ĐH Ngoại thương
PLĐC-chương 1 (1).ppt của trường  ĐH Ngoại thươngPLĐC-chương 1 (1).ppt của trường  ĐH Ngoại thương
PLĐC-chương 1 (1).ppt của trường ĐH Ngoại thương
 
Cau-Trắc-Nghiệm-TTHCM-Tham-Khảo-THI-CUỐI-KI.pdf
Cau-Trắc-Nghiệm-TTHCM-Tham-Khảo-THI-CUỐI-KI.pdfCau-Trắc-Nghiệm-TTHCM-Tham-Khảo-THI-CUỐI-KI.pdf
Cau-Trắc-Nghiệm-TTHCM-Tham-Khảo-THI-CUỐI-KI.pdf
 
Văn 7. Truyện ngụ ngôn Rùa và thỏ+ Viết PT nhân vật.docx
Văn 7. Truyện ngụ ngôn Rùa và thỏ+ Viết PT nhân vật.docxVăn 7. Truyện ngụ ngôn Rùa và thỏ+ Viết PT nhân vật.docx
Văn 7. Truyện ngụ ngôn Rùa và thỏ+ Viết PT nhân vật.docx
 
BÁO CÁO CUỐI KỲ PHÂN TÍCH THIẾT KẾ HƯỚNG ĐỐI TƯỢNG - NHÓM 7.docx
BÁO CÁO CUỐI KỲ PHÂN TÍCH THIẾT KẾ HƯỚNG ĐỐI TƯỢNG - NHÓM 7.docxBÁO CÁO CUỐI KỲ PHÂN TÍCH THIẾT KẾ HƯỚNG ĐỐI TƯỢNG - NHÓM 7.docx
BÁO CÁO CUỐI KỲ PHÂN TÍCH THIẾT KẾ HƯỚNG ĐỐI TƯỢNG - NHÓM 7.docx
 
trắc nhiệm ký sinh.docxddddddddddddddddd
trắc nhiệm ký sinh.docxdddddddddddddddddtrắc nhiệm ký sinh.docxddddddddddddddddd
trắc nhiệm ký sinh.docxddddddddddddddddd
 

Chuyên đề phương tích và ứng dụng

  • 1. TRẠI HÈ HÙNG VƯƠNG LẦN THỨ IX CHUYÊN ĐỀ PHƯƠNG TÍCH VÀ ỨNG DỤNG Nguyễn Quỳnh, Chuyên Hùng Vương, Phú Thọ Đào Văn Lương, Chuyên Lào Cai Hoàng Thông, Chuyên Lê Quý Đôn, Điện Biên HÒA BÌNH, THÁNG 8 NĂM 2013
  • 2. Trang 1 MỤC LỤC Trang Phần A Cơ sở lý thuyết 2 1. Phương tích của một điểm đối với một đường tròn 2 2. Trục đẳng phương của hai đường tròn 3 3. Tâm đẳng phương của ba đường tròn 5 Phần B Ứng dụng phương tích giải một số bài tập hình học phẳng 7 1. Các bài tập sử dụng tính chất của phương tích 7 2. Các bài tập sử dụng tính chất của trục đẳng phương 10 3. Các bài tập sử dụng tính chất của tâm đẳng phương 23 Phần C Bài tập đề nghị 28 1. Đề bài 28 2. Lời giải 30 Tài liệu tham khảo 40
  • 3. Trang 2 PHẦN A: CƠ SỞ LÝ THUYẾT 1. Phương tích của một điểm đối với một đường tròn 1.1 Bài toán Cho đường tròn (O; R) và điểm M cố định, OM = d. Một đường thẳng thay đổi qua M cắt đường tròn tại hai điểm A và B. Khi đó MA.MB = MO2 − R2 = d 2 − R2. Chứng minh B C M O A Gọi C là điểm đối xứng của A qua O. Ta có CB ⊥ AM hay B là hình chiếu của C trên AM. ( OC)( OA) Khi đó ta có MA.MB = MA.MB = MC.MA = MO +MO + ( )( ) 2 2 = − + = − = − = − MO OA MO OA MO OA 2 2 2 2. OM OA d R 1.2 Định nghĩa Đại lượng không đổi MA.MB = d 2 − R2 trong Bài toán 1.1 được gọi là phương tích của điểm M đối với đường tròn (O), kí hiệu PM/(O). Ta có: ( ) 2 2 / . M O P = MAMB = d − R . 1.3 Tính chất 1.3.1 Tính chất 1 Điểm M nằm bên ngoài đường tròn (O) khi và chỉ khi /( ) 0. M O P Điểm M nằm trên đường tròn (O) khi và chỉ khi /( ) 0. M O P = Điểm M nằm bên trong đường tròn (O) khi và chỉ khi /( ) 0. M O P
  • 4. 1.3.2 Tính chất 2 Trong mặt phẳng, cho đường tròn (O;R) và một điểm M nằm bên ngoài (O). Qua M kẻ cát tuyến MAB và tiếp tuyến MT tới (O). Khi đó Trang 3 MA.MB = MT 2 = OM2 − R2. 1.3.3 Tính chất 3 Cho hai đường thẳng AB,CD phân biệt cắt nhau tại M (M không trùng A,B, C,D). Khi đó, nếu MA.MB = MC.MD thì bốn điểm A,B,C,D cùng nằm trên một đường tròn. 1.3.4 Tính chất 4 Cho hai đường thẳng AB,MT phân biệt cắt nhau tại M (M không trùng A,B,T ). Khi đó, nếu MA.MB = MT 2 thì đường tròn ngoại tiếp tam giác ABT tiếp xúc với MT tại T. 1.4 Phương tích trong hệ tọa độ Descartes Trong mặt phẳng, với hệ tọa độ Descartes, cho điểm ( ) 0 0 M x ; y và đường tròn (C) : x2 + y2 + 2ax + 2by + c = 0. Đặt F(x; y) = x2 + y2 + 2ax + 2by + c, khi đó P 2 2 ( ) M / O1 0 0 0 0 0 0 = F(x ; y ) = x + y + 2ax + 2by + c. 2. Trục đẳng phương của hai đường tròn 2.1 Định lý và định nghĩa Cho hai đường tròn không đồng tâm (O1; R1) và (O2; R2). Tập hợp các điểm M có phương tích đối với hai đường tròn bằng nhau là một đường thẳng, đường thẳng này được gọi là trục đẳng phương của hai đường tròn (O1) và (O2). Chứng minh Giả sử điểm M có cùng phương tích đối với hai đường tròn đã cho. Gọi H là hình chiếu của M trên O1O2, I là trung điểm của O1O2. Ta có: ( 2 2 ) ( 2 2 ) 2 2 ⇔ + − + = − ⇔ − = − ( )( ) 2 2 MH HO MH HO R R 1 2 1 2 2 2 2 2 1 2 1 2 . HO HO R R ⇔ HO1 − HO2 HO1 + HO2 = R1 − R2
  • 5. Trang 4 M I O2 O1 H 2 2 − R R 2 2 1 2 .2 . 2 1 1 2 1 2 2 O O HI R R IH OO ⇔ = − ⇔ = Do H cố định, suy ra tập hợp các điểm M có cùng phương tích đối với hai đường tròn là đường thẳng qua H và vuông góc với O1O2. 2.2 Tính chất Cho hai đường tròn (O1) và (O2). Từ định lý 2.1 ta có các tính chất sau: 2.2.1 Tính chất 1 Trục đẳng phương của hai đường tròn vuông góc với đường thẳng nối tâm. 2.2.2 Tính chất 2 Nếu hai đường tròn cắt nhau tại A và B thì AB chính là trục đẳng phương của chúng. 2.2.3 Tính chất 3 Nếu điểm M có cùng phương tích đối với (O1) và (O2) thì đường thẳng qua M vuông góc với O1O2 là trục đẳng phương của hai đường tròn. 2.2.4 Tính chất 4 Nếu hai điểm M, N có cùng phương tích đối với hai đường tròn thì đường thẳng MN chính là trục đẳng phương của hai đường tròn. 2.2.5 Tính chất 5 Nếu 3 điểm có cùng phương tích đối với hai đường tròn thì 3 điểm đó thẳng hàng. 2.2.6 Tính chất 6
  • 6. Nếu (O1) và (O2) tiếp xúc nhau tại A thì đường thẳng qua A và vuông góc với O1O2 chính là trục đẳng phương của hai đường tròn. 2.2 Cách xác định trục đẳng phương của hai đường tròn không đồng tâm Trong mặt phẳng cho hai đường tròn không đồng tâm (O1) và (O2). Xét các trường hợp sau: 2.2.1 Trường hợp 1: Hai đường tròn cắt nhau tại hai điểm phân biệt A, B. Khi đó đường thẳng AB chính là trục đẳng phương của hai đường tròn. 2.2.2 Trường hợp 2: Hai đường tròn tiếp xúc nhau tại T. Khi đó tiếp tuyến chung tại T chính là trục đẳng phương của hai đường tròn. 2.2.3 Trường hợp 3: Hai đường tròn không có điểm chung. Dựng đường tròn 3 (O ) cắt cả hai đường tròn. Trục đẳng phương của các cặp đường tròn 1 3 (O )và (O ); 2 3 (O )và (O ) cắt nhau tại K. Đường thẳng qua K vuông góc với O1O2 là trục đẳng phương của Trang 5 1 2 (O ),(O ). 2.3 Trục đẳng phương trong Hệ tọa độ Descartes Trong mặt phẳng tọa độ Oxy cho các đường tròn không đồng tâm: (C ) : x 2 + y 2 + 2a x + 2b y + c = 0 , (C ) : x 2 + y 2 + 2a + 2b + = 0 1 1 1 1 2 2 x 2 y c 2 Từ biểu thức phương tích của một điểm đối với một đường tròn trong hệ tọa độ suy ra trục đẳng phương của 1 (C ) và 2 (C ) là đường thẳng có phương trình ( ) ( ) ( ) 1 2 1 2 1 2 2 a − a x + 2 b − b y + c − c = 0 Δ 3. Tâm đẳng phương của ba đường tròn 3.1 Định lý và định nghĩa Cho 3 đường tròn (C1), (C2) và (C3). Khi đó 3 trục đẳng phương của các cặp đường tròn này hoặc trùng nhau hoặc song song hoặc cùng đi qua một điểm. Nếu các trục đẳng phương cùng đi qua một điểm thì điểm đó được gọi là tâm đẳng phương của ba đường tròn. Chứng minh Gọi dij là trục đẳng phương của hai đường tròn (Ci) và (Cj). Ta xét hai trường hợp sau. a)Giả sử có một cặp đường thẳng song song, không mất tính tổng quát ta giả sử d12 // d23. Ta có 12 1 2 23 2 3 d ⊥ OO , d ⊥ O O suy ra 1 2 3 O ,O ,O thẳng hàng. Mà d13 ⊥ O1O3 suy ra 13 23 12 d // d // d
  • 7. Trang 6 b)Giả sử d12 và d23 có điểm chung M. Khi đó ta có P = M / ( P O ) M / ( O )  ⇒ = ⇒ ∈  ( ) ( ) ( ) ( ) 1 2 1 3 2 3 / / 13 / / M O M O M O M O M d =  P P P P d12 d13 d23 M O1 O2 O3 Từ đây suy ra nếu có hai đường thẳng trùng nhau thì đó cũng là trục đẳng phương của cặp đường tròn còn lại. Nếu hai trục đẳng phương chỉ cắt nhau tại một điểm thì điểm đó cũng thuộc trục đẳng phương còn lại 3.2 Tính chất 3.2.1 Tính chất 1: Nếu 3 đường tròn đôi một cắt nhau thì các dây cung chung cùng đi qua một điểm 3.2.2 Tính chất 2: Nếu 3 trục đẳng phương song song hoặc trùng nhau thì tâm của 3 đường tròn thẳng hàng. 3.2.3 Tính chất 3: Nếu 3 đường tròn cùng đi qua một điểm và có các tâm thẳng hàng thì các trục đẳng phương trùng nhau.
  • 8. Trang 7 PHẦN B: ỨNG DỤNG PHƯƠNG TÍCH GIẢI MỘT SỐ BÀI TẬP HÌNH HỌC PHẲNG 1. CÁC BÀI TẬP SỬ DỤNG TÍNH CHẤT CỦA PHƯƠNG TÍCH Bài tập 1.1 (S44 Mathematical Reflection MR2-2007) Từ một điểm P nằm bên ngoài đường tròn tâm O, kẻ các tiếp tuyến PA, PB tới đường tròn (O), (A, B là các tiếp điểm). Gọi M là trung điểm AP và N là giao điểm của BM với (O), (N không trùng B). Chứng minh rằng PN = 2MN. Lời giải Ta có A P O MN.MB = MA2 = MA.MP . Gọi N ' là điểm đối xứng với N qua M, khi đó MN.MB = MN '.MB, suy ra MA.MP = MN 'MB, hay tứ giác ABPN ' là tứ giác nội tiếp một đường tròn. Đặt NAP = α,NAB = β, ta có PAN ' = PBN ' = BAN = β⇒ NAN ' =ANN ' = α + β. Mặt khác ANN ' = NAB + NBA = α + β, hay tam giác NPN’ cân tại N suy ra PN = 2MN. Bài tập 1.2 N' N M B
  • 9. Cho đường tròn (O) và hai điểm A, B cố định. Một đường thẳng quay quanh A, cắt (O) tại M và N. Chứng minh rằng tâm đường tròn ngoại tiếp tam giác BMN thuộc một đường thẳng cố định. Lời giải Trang 8 C M N O I A B Gọi I là tâm đường tròn ngoại tiếp tam giác MNB. AB cắt (I) tại điểm thứ hai C. Ta có A I A O = AC AB = AM AN = (không đổi vì A, (O) cố định). /( ) /( ) P . . P P Suy ra A /( O ) AC AB = Vì A, B cố định và C thuộc AB nên từ hệ thức trên ta có C cố định. Suy ra I thuộc đường trung trực của BC cố định. Suy ra AB.AC AK = là hằng số nên điểm K cố đinh. Bài tập được chứng minh AI Bài tập 1.3 Cho đường tròn (O) và dây AB. Trên tia AB lấy điểm C nằm ngoài đường tròn (O). Từ điểm E chính giữa cung lớn AB kẻ đường kính EF cắt dây AB tại D. Tia CE cắt (O) tại điểm I. Cho A, B, C cố định, chứng minh rằng khi đường tròn (O) thay đổi nhưng vẫn đi qua A, B thì đường FI luôn đi qua một điểm cố định. Lời giải
  • 10. Trang 9 E ⇒ = ⇒ điểm K cố định A C K H I B O N M K I F A D B C Gọi K là giao điểm của FI và AB Tứ giác DKIE nội tiếp đường tròn đường kính EK nên /[ ] /( ) . . . C EK C O P = CK CD = CI CE = P = CB CA CA.CB CK CD Vậy đường thẳng FI luôn đi qua một điểm K cố định Bài tập 1.4 Cho ba điểm cố định A, B, C thẳng hàng (theo thứ tự đó). Một đường tròn (O) thay đổi nhưng luôn đi qua B, C. Từ điểm A kẻ các tiếp tuyến AM, AN đến (O). Đường thẳng MN cắt AO và AC lần lượt tại H và K. Gọi I là trung điểm BC. Chứng minh rằng đường tròn ngoại tiếp tam giác OHI luôn đi qua hai điểm cố định Lời giải Hiển nhiên điểm I cố định. Do HOIK nội tiếp nên ta có [ ] 2 / . . A KO P = AK AI = AH AO = AM (do tam giác AMO vuông tại M có MH là đường cao) Ta lại có AM là tiếp tuyến của (O) nên ( ) AM 2 = P = AB . AC . A / O
  • 11. Trang 10 2. CÁC BÀI TẬP SỬ DỤNG TÍNH CHẤT CỦA TRỤC ĐẲNG PHƯƠNG Bài tập 2.1 (IMO 2013 Problem 4) Cho tam giác nhọn ABC với trực tâm H. Cho W là một điểm tùy ý trên cạnh BC, khác với các điểm B và C. Các điểm M và N tương ứng là chân các đường cao hạ từ B và C. Kí hiệu 1 ω là đường tròn ngoại tiếp tam giác BWN, và gọi X là điểm trên 1 ω sao cho WX là đường kính của 1 ω . Tương tự, kí hiệu 2 ω là đường tròn ngoại tiếp tam giác CWM, và gọi Y là điểm trên 2 ω sao cho WY là đường kính của 2 ω . Chứng minh rằng các điểm X,Y và H thẳng hàng. Lời giải w2 w1 Z Y O2 X O1 P M N A H B C W Gọi P là chân đường cao kẻ từ A của tam giác ABC, O ,O là tâm các đường tròn ω ,ω , 1 2 1 2 Z là giao điểm thứ hai của ω và ω . 1 2 Tứ giác BNMC nội tiếp đường tròn đường kính BC nên AN.AB = AM.AC hay A thuộc trục đẳng phương của ω và ω . Suy ra A, Z, W cùng nằm trên một đường thẳng vuông 1 2góc với OO và XY (1) 1 2 Tứ giác BNHP nội tiếp nên AH.AP = AN.AB = AZ.AW, từ đó PHZW là tứ giác nội tiếp hay HZ vuông góc với ZW (2)
  • 12. Từ (1) và (2) suy ra X, Y, H thẳng hàng, điều phải chứng minh. Bài tập 2.2 Cho tứ giác ABCD nội tiếp đường tròn (O), AB ≠ CD. Dựng hai hình thoi AEDF và BMCN có cạnh bằng nhau. Chứng minh bốn điểm E, F, M, N cùng thuộc một đường tròn. Lời giải Dựng (A,AF) và (B,BM) . Do OA = OB và AF = BM nên O nằm trên trục đẳng phương của (A) và (B). Trang 11 M N E O A D C B F Mặt khác, EF, MN lần lượt là trung trực của đoạn AD, BC nên EF ∩MN = {O} ⇒OE.OF = OM.ON Suy ra 4 điểm E, F, M, N cùng thuộc một đường tròn (đpcm). Bài tập 2.3 Cho hai đường tròn ngoài nhau (O1), (O2 ). Kẻ tiếp tuyến chung ngoài A1A2 của hai đường tròn (A1∈(O1); A2 ∈(O2 )). Gọi K là trung điểm của A1A2 , từ K kẻ các tiếp tuyến KB1,KB2 tới (O1) và (O2 ). A1B1, A2B2 cắt nhau tại L, KL cắt O1O2 tại P . Chứng minh rằng B1,B2,P,L cùng nằm trên một đường tròn. Lời giải Do KA1 = KA2 = KB1 = KB2 nên tứ giác A1B1B2A2 nội tiếp
  • 13. ⇒ LB1.LA1 = LB2.LA2 Suy ra KL là trục đẳng phương của ( ) O1 và ( ) O2 ⇒ KL ⊥ O1O2 3 điểm A1,B1,P nhìn đoạn O1K dưới góc 900 nên tứ giác A1B1PK nội tiếp, tương tự tứ giác A2B2PK nội tiếp Bài tập 2.4 (IMO 1995) Trên đường thẳng d lấy bốn điểm A, B, C, D theo thứ tự đó. Các đường tròn đường kính AC và BD cắt nhau tại X và Y. Đường thẳng XY cắt BC tại Z. Trên đường thẳng XY lấy một điểm P không trùng với Z, đường thẳng CP cắt đường tròn đường kính AC tại điểm thứ hai M, đường thẳng BP cắt đường tròn đường kính BD tại điểm thứ hai N. Chứng minh AM, DN, XY đồng quy. Lời giải Gọi J , J ′,Z lần lượt là giao của AM, DN, AD với XY Tứ giác JMCK nội tiếp nên PJ.PK = PM.PC Tương tự PJ′.PK = PN.PB Do P nằm trên trục đẳng phương của đường tròn đường kính AC và đường tròn đường kính BD nên PM.PC = PN.PB⇒ PJ.PK = PJ′.PK hay P ≡ P′ Vậy AM, DN, XY đồng quy (đpcm). Bài tập 2.5 Cho tam giác ABC, các đường cao AD, BE, CF đồng quy tại H, M là trung điểm BC, EF cắt BC tại I, J là trung điểm của MH. Chứng minh IH vuông góc với OJ. Lời giải Gọi O, J lần lượt là trung điểm AH, MH. Ta có ∠EFD = 2∠HFD = 2∠EBM = ∠EMC Suy ra tứ giác FEMD nội tiếp. ⇒ IE.IF = IM.ID Do đó I nằm trên trục đẳng phương của (O,OA) và ( J , JH ) Trang 12
  • 14. ⇒IH ⊥ OJ Mà OJ là đường trung bình của tam giác AMH nên OJ//AM ⇒IH ⊥ AM (đpcm) Bài tập 2.6 Cho tam giác ABC nội tiếp trong đường tròn (O). AA’, BB’, CC’ là các đường cao của tam giác. Kí hiệu (W ) A là đường tròn đi qua A, A’ và tiếp xúc với OA. (W ),(W ) B C được định nghĩa tương tự. Chứng minh rằng ba đường tròn đó cắt nhau tại hai điểm thuộc đường thẳng Euler của tam giác ABC. Lời giải Ta có / (W ) . . / (W ) PH A = HA HA′ = HC HC′ = PH C và do OA, OC lần lượt tiếp xúc với W ,W A C nên / (W ) 2 2 / (W ) PO A = OA = OC = PO C Suy ra OH là trục đẳng phương của (W ) A và (W ) C (W ) (W ) { , } ⇒ A ∩ B = E F ∈HO Vậy 3 đường tròn (W ) A , (W ) B , (W ) C cắt nhau tại 2 điểm thuộc đường thẳng Ơ-le của tam giác ABC (đpcm). Bài tập 2.7 Cho tam giác không cân ABC nội tiếp đường tròn (O) và ngoại tiếp đường tròn (I). Các điểm A’, B’, C’ lần lượt thuộc các đường thẳng BC, CA, AB sao cho ' ' ' 90o. AIA = BIB = CIC = Chứng minh A’, B’, C’ cùng thuộc một đường thẳng và đường thẳng đó vuông góc với OI. Lời giải Kí hiệu (I, 0) là đường tròn tâm I, bán kính bằng 0. Trang 13 Ta có 900 ABC IBC = = AIC − = CIA′ 2 2 . ⇒ IA′ = A′B A′C hay PA/(I ;0) = PA′/(O) Tương tự PB/(I ;0) = PB′/(O) , PC/(I ;0) = PC'/(O)
  • 15. Suy ra A′,B′,C′ cùng thuộc trục đẳng phương của (O) và (I,0), đường thẳng này vuông góc với OI (đpcm). Bài tập 2.8 Cho đường tròn tâm O đường kính AB, và điểm H cố định thuộc AB. Từ điểm K thay đổi trên tiếp tuyến tại B của O, vẽ đường tròn (K; KH) cắt (O) tại C và D. Chứng minh rằng CD luôn đi qua một điểm cố định. Lời giải Trang 14 I M D C B A O K H Gọi I là điểm đối xứng của H qua B, suy ra I cố định và thuộc (K). Gọi M là giao điểm của CD và AB. Vì CD là trục đẳng phương của (O) và (K) nên ta có: = = . . . MH MI MC MD MAMB MB BH MB BI MB MB BA ( )( ) ( ) ( )( ) 2 ⇔ + + = + ⇔ + − = + ⇔ − = + ⇔ = MB BH MB BH MB MB BA 2 2 2 2 . . MB BH MB MB BA BH BM BA Vì A, B, H cố định suy ra M cố định.
  • 16. Bài tập 2.9 Cho tam giác ABC có đỉnh A cố định và B, C thay đổi trên đường thẳng d cố định sao cho nếu gọi A’ là hình chiếu của A lên d thì A′B.A′C âm và không đổi. Gọi M là hình chiếu của A’ lên AB; N là hình chiếu của A’ lên AC; K là giao điểm của các tiếp tuyến của đường tròn ngoại tiếp tam giác A’MN tại M và N. Chứng minh rằng K thuộc một đường thẳng cố định. Lời giải Trang 15 I P D H K M N A B A' C Gọi O là tâm đường tròn ngoại tiếp tam giác A’MN và I là giao điểm của OK và MN. Ta thấy O chính là trung điểm của AA’. Dễ thấy 2 AM.AB = AA′ = AN.AC Suy ra tứ giác BMNC nội tiếp. ⇒ AMN = ACB Mà ADB = ACBNên AMN = ADB Suy ra MPDB nội tiếp. Do đó ta có 2 AP.AD = AM.AB = AA′ Mà A, A’ và D cố định suy ra P cố định. 1 Gọi H là hình chiếu của K trên AA’.Ta có AP . AH = AI . AK = IN 2 = AA′ 2 4 Mà A, P, A’ cố định suy ra H cố định. Vậy K thuộc đường thẳng qua H và vuông góc với AA’. Bài tập 2.10 Trên đường thẳng d lấy 4 điểm A, B, C, D (theo thứ tự đó). Đường tròn đường kính AC và BD cắt nhau tại X, Y. Đường thẳng XY cắt BC tại Z. Lấy P là một điểm trên XY khác Z.
  • 17. Đường thẳng CP cắt đường tròn đường kính AC tại điểm thứ hai là M, và BP cắt đường tròn đường kính BD tại điểm thứ hai là N. Chứng minh rằng AM, DN và XY đồng qui. Lời giải Gọi Q, Q’ lần lượt là giao điểm của DN và AM với XY. Ta cần chứng minh Q ≡ Q′ . Tứ giác QMCZ nội tiếp, suy ra PM.PC = PQ.PZ Tứ giác NQ’ZB nội tiếp, suy ra PQ′.PZ = PN.PB Mà P thuộc XY là trục đẳng phương của đường tròn đường kính AC và đường tròn đường kính BD nên PN.PB = PX.PY = PM.PC Trang 16 Q P X A B Z C D N M Y Suy ra PQ.PZ = PQ′.PZ ⇒Q ≡ Q′ Vậy XY, AM và DN đồng quy. Bài tập 2.11 Cho H là trực tâm tam giác ABC không cân góc A nhọn; Hình chiếu vuông góc của H trên AB, AC theo thứ tự là E, F. Gọi D là trung điểm BC; P, Q là giao điểm của hai đường tròn đường kính AD và BC. Chứng minh H, P, Q thẳng hàng và các đường thẳng BC, EF, PQ đồng quy. Lời giải Gọi G là chân đường cao kẻ từ đỉnh A của tam giác ABC. Ta có /[ ] /[ ] . . H BC H AD P = HE HC = HG HA = P suy ra H nằm trên trục đẳng phương của hai đường tròn đường kính BC và AD. Suy ra H, P, Q thẳng hàng.
  • 18. Gọi I là giao điểm của EF và BC. (DEF) là đường tròn Euler của tam giác ABC nên G nằm trên (DEF). Do đó /[ ] /[ ] . . . I BC I AD P = IB IC = IE IF = IG ID = P Suy ra I, P, G thẳng hàng hay BC, EF, PQ đồng quy tại I. Bài tập 2.12 Cho tam giác ABC có trực tâm H. Đường tròn đi qua B, C cắt AB, AC tại D, E. Gọi F là trực tâm tam giác ADE và I là giao điểm của BE và CD. Chứng minh I, H, F thẳng hàng Lời giải Gọi F1, F2 là hình chiếu vuông góc của F lên AB, AC; H1,H2 là hình chiếu vuông góc của H lên AB, AC Ta có 1 2 1 2 FF.FE = FF .FD; HH .HC = HH .HB (tính chất trực tâm) Mặt khác ta lại có IE.IB = IC.ID Suy ra F, H, I cùng thuộc trục đẳng phương của hai đường tròn đường kính BD và CE nên chúng thẳng hàng Bài tập 2.13 Cho tứ giác ABCD có các cạnh AB và CD cắt nhau tại I, AD và BC cắt nhau tại K. 1) Chứng minh rằng trực tâm của các tam giác AID, ABK, BCI, CDK thẳng hàng 2) Chứng minh rằng trung điểm của các đoạn AC, BD, IK thẳng hàng. Lời giải 1)Gọi O1; O2; O3 lần lượt là trung điểm các đoạn AC, BD, IK Các đường cao của tam giác CDK lần lượt là CC', DD'; KK' và H là trực tâm tam giác Trang 17
  • 19. Trang 18 K' A C' K D' H O1 O2 O3 I D B C Ta có ( ) ( ) / 1 / 2 . '; . ' H O H O P = HC HC P = HD HD xét đường tròn đường kính CD ta lại có HC.HC' = HD.HD' nên H thuộc trục đẳng phương của hai đường tròn (O1),(O2) Chứng minh tương tự cho trực tâm các tam giác AID, ABK, BCI cũng có cùng phương tích với hai đường tròn (O1),(O2). Suy ra điều phải chứng minh 2)Tương tự ta chứng minh được H có cùng phương tích với hai đường tròn (O2) và (O3) Gọi J là trực tâm của tam giác BCI. Khi đó ta cũng chứng minh được J có cùng phương tích với ba đường tròn (O1),(O2), (O3). Suy ra ba đường tròn này đôi một nhận HJ làm trục đẳng phương suy ra đpcm Bài tập 2.14 Cho đường tròn tâm O và hai đường kính AB, CD. Tiếp tuyến với (O) tại B cắt AC tại P, PD cắt (O) tại điểm thứ hai G. Chứng minh rằng AG, BC, PO đồng quy. Lời giải Gọi I, I' lần lượt là trung điểm PA, PC. Ta có bốn điểm O, B, C, I thuộc đường tròn (T) đường kính BI Ta chứng minh được GAI ' = CDG = COI ' = GOI ' suy ra bốn điểm O, A, G, I' cùng thuộc đường tròn (T') khi đó /( ) /( ') 0 O T O T P = P = Hơn nữa /( ) /( ') . . ' P T P T P = PI PC = PA PI = P
  • 20. Trang 19 j F I O H I I' E G P N M D A B C B C O A D suy ra OP là trục đẳng phương của (T) và (T') Nhưng AG là trục đẳng phương của (O) và (T'), BC là trục đẳng phương của (O) và (T) Vậy AG, BC, PO đồng quy. Bài tập 2.15 Cho tam giác ABC có đường cao BD và CE cắt nhau tai H. M là trung điểm của BC, N là giao điểm của DE và BC. Chứng minh rằng NH vuông góc với AM. Lời giải Ta có = = = DEH DAH DBC FEH 2. 2. ⇒ = = = FED FEH DBC DMC Suy ra tứ giác EDMF nội tiếp. Từ đó ta có NE.ND = NF.NM , suy ra N nằm trên trục đẳng phương của đường tròn đường kính MH và đường tròn đường kính AH. Mặt khác H là giao điểm của (O) và (I), suy ra NH chính là trục đẳng phương của (O) và (I). Suy ra NH ⊥ OI , rõ ràng OI // AM, do đó NH ⊥ AM . Bài tập 2.16 Cho tam giác ABC. Một đường thẳng song song với BC cắt AB, AC tại D và E. Gọi P là một điểm bên trong tam giác ADE, F và G là giao của DE với BP và CP. Đường tròn tâm
  • 21. (O) ngoại tiếp tam giác PDG, đường tròn tâm (I) ngoại tiếp tam giác PEF cắt nhau tại điểm thứ hai là Q. Chứng minh rằng AQ ⊥ OI Lời giải Gọi M là giao điểm thứ hai của AB và (PDG), N là giao thứ hai của AC và (PFG) Trang 20 M N P F G E A D B C Ta có AMP = PGD và PGD = PCB (đồng vị), suy ra AMP = PCB , suy ra BMPC nội tiếp. Chứng minh tương tự PNCB nội tiếp. Suy ra BMNC nội tiếp, suy ra AM.AB = AN.AC Mà AD AE = (Định lý Thalet). Suy ra AM.AD = AN.AE AB AC Do đó A thuộc trục đẳng phương PQ của (PDG) và (PEF) suy ra AQ ⊥ OI . Bài tập 2.17 Cho tam giác ABC không cân nội tiếp (O) và ngoại tiếp (I). Các đường thẳng qua I vuông góc với AI, BI, CI cắt BC, CA, AB tại M, N, P theo thứ tự. Chứng minh M, N, P cùng nằm trên một đường thẳng vuông góc với OI. Lời giải
  • 22. Trang 21 Giả sử AB AC khi đó M nằm trên tia đối của tia BC. Ta có C C AIB = 90 0 + ⇒MIB = = MCI 2 2 Suy ra ΔMIB ∼ ΔMCI ⇒MI 2 = MB.MC Suy ra M nằm trên trục đẳng phương của (O) và đường tròn tâm I. Tương tự với N, P ta suy ra M, N, P thẳng hàng và đường thẳng đi qua ba điểm đó vuông góc với OI. Bài tập 2.18 Cho tam giác ABC nhọn. Các đường cao BE, CF cắt nhau tại H. Trên các tia FB, EC lấy các điểm P, Q theo thứ tự sao cho FP = FC, EQ = EB. PQ cắt CP tại K, I, J theo thứ tự tà trung điểm BQ, CP. Chứng minh HK vuông góc với IJ. Lời giải Từ giả thiết ta có BPC = BQC = 450 suy ra tứ giác BCQP nội tiếp /[ ] /[ ] . . K BQ K CP ⇒ P = KB KQ = KC KP = P Theo tính chất trực tâm tam giác tac có /[ ] /[ ] . . H BQ H CP P = HB HE = HC HF = P vậy HK là trục đẳng phương của hai đường tròn đường kính BQ và CP và IJ là đường nối tâm của hai đường nối tâm của hai đường tròn nên HK ⊥ IJ
  • 23. Bài tập 2.19 Cho hai đường tròn ngoài nhau (O1), (O2 ). Kẻ các tiếp tuyến chung ngoài A1A2 và chung trong B1B2 của hai đường tròn (A1,B1∈(O1); A2,B2 ∈(O2 )). Chứng minh A1B1, A2B2, O1O2 đồng quy. Lời giải Gọi M là giao điểm của A1B1 với A2B2 . Dễ dàng có A1B1 ⊥ A2B2 Gọi (C1),(C2 ) lần lượt là các đường tròn đường kính A1A2 , B1B2 Do 0 ∠A1MA2 = ∠B1MB2 = 90 nên M nằm trên trục đẳng phương của (C1) và (C2 ) . O1A1 = O1B1 và O1A1,O1B1 lần lượt là tiếp tuyến của (C1),(C2 ) nên O1 nằm Trang 22 Mặt khác 2 2 trên trục đẳng phương của (C1) và (C2 ) . Tương tự O2 cũng nằm trên trục đẳng phương của (C1) và (C2 ) Suy ra O1,M,O2 thẳng hàng.
  • 24. 3. CÁC BÀI TẬP SỬ DỤNG TÍNH CHẤT CỦA TÂM ĐẲNG PHƯƠNG Bài tập 3.1 Cho đường tròn (O) đường kính AB và CD. Tiếp tuyến tại B của (O) cắt AC tại E, DE cắt (O) tại điểm thứ hai F. Chứng minh rằng AF, BC, OE đồng quy. Lời giải Trang 23 F E B D O A C Kí hiệu (C1),(C2 ) lần lượt là đường tròn ngoại tiếp tam giác AEF, BCE. Ta có AF, BC là trục đẳng phương của (O) và (C1), (O) và (C2 ) . Mặt khác OAF = FDB = FEA,OBC = CEB Suy ra OA, OB lần lượt là tiếp tuyến của (C1),(C2 ) và lại có 2 2 OA = OB Do đó OE là trục đẳng phương của (C1) và (C2 ) . Suy ra AF, BC, OE đồng quy tại tâm đẳng phương của ba đường tròn. Bài tập 3.2 (Iran MO 1996) Trên hai cạnh AB, AC của tam giác ABC lần lượt lấy các điểm D, E sao cho DE song song với BC. Gọi P là điểm tùy ý bên trong tam giác ABC, các đường thẳng PB, PC cắt DE tại F và G. Gọi 1 2 O ,O lần lượt là tâm các đường tròn ngoại tiếp các tam giác PDG, PEF. Chứng minh rằng AP vuông góc với 1 2OO .
  • 25. Trang 24 Lời giải N M F G O2 O1 E A B C P D Gọi M là giao điểm thứ hai của AB với (O1), N là giao điểm thứ hai của AC với (O2 ) Suy ra tứ giác B, P, C, M nằm trên một đường tròn, tương tự B, P, C, N nằm trên một đường tròn do đó tứ giác BMNC nội tiếp. Mà DE//BC ta thu được tứ giác MDEN nội tiếp. Áp dụng định lý về tâm đẳng phương cho các đường tròn ngoại tiếp DGP, PEF và DENM ta có DM ∩EN = {A} nằm trên trục đẳng phương của (O1) và (O2 ) Suy ra AP ⊥ O1O2 (đpcm). Bài tập 3.3 Cho nửa đường tròn đường kính AB và một diểm C nằm trên nửa đường tròn đó. Gọi H là hình chiếu của C trên AB. Đường tròn đường kính CH cắt CA tại E, CB Tại F và đường tròn đường kính AB tại D. Chứng minh rằng CD, EF, AB đồng quy. Lời giải Do ACB = 900 nên EF là đường kính của đường tròn đường kính CH ⇒CEF =ACH = CBA Suy ra tứ giác AEFB nội tiếp.
  • 26. Áp dụng định lý về tâm đẳng phương cho đường tròn ngoại tiếp tứ giác AEFB, đường tròn đường kính AB và đường kính EF ta có CD, EF, AB đồng quy (đpcm). Bài tập 3.4 Cho tam giác ABC nội tiếp đường tròn ( );( ),( ) O I Ia lần lượt là đường tròn nội tiếp và bàng tiếp góc A của tam giác ABC. Giả sử IIa cắt BC và (O) lần lượt tại A′ ,M . Gọi N là trung điểm cung MBA. , NI NIa cắt (O) lần lượt tại S,T. Chứng minh rằng S,T, A′ thẳng hàng. Lời giải Ta có: Trang 25 ( ) ( ) 1 1 NST = sd NA + sd AS = sd NM + sd AS = NIM 2 2 Suy ra IaTS = IaIS Suy ra tứ giác IaTI nội tiếp đường tròn (w1) Mặt khác 900 IBIa = ICIa = nên tứ giác IBIaC nội tiếp đường tròn (w2 ). Ta thấy IIa là trục đẳng phương của (w1) và (w2 ), BC là trục đẳng phương của (O) và (w2 ), TS là trục đẳng phương của (O) và (w1) . Theo định lý về tâm đẳng phương thì , , IIa TS BC đồng quy tại A′ . Vậy T, A′,S thẳng hàng (đpcm). Bài tập 3.5 Bên ngoài tam giác ABC vẽ các tam giác cân BCD, CAE, ABF có các cạnh đáy tương ứng là BC, CA, AB. Chứng minh rằng các đường thẳng lần lượt đi qua A, B, C và vuông góc với EF, FD, DE tương ứng đồng quy. Lời giải Gọi C1,C2,C3 lần lượt là đường tròn tâm D, bán kính DB, đường tròn tâm E, bán kính EA, đường tròn tâm F bán kính FA.
  • 27. Đường thẳng qua C vuông góc với DE là trục đẳng phương của (C1) và (C2 ), đường thẳng qua A vuông góc với EF là trục đẳng phương của (C2 ) và (C3 ) , đường thẳng qua B vuông góc với DF là trục đẳng phương của (C1) và (C3 ) . Do đó 3 đường thẳng này đồng quy tại tâm đẳng phương của 3 đường tròn (đpcm). Bài tập 3.6 Cho tam giác ABC, các điểm A’, B’ lần lượt nằm trên hai cạnh BC và CA. Chứng minh rằng trục đẳng phương của hai đường tròn đường kính AA’ và BB’ đi qua trực tâm H của tam giác ABC. Lời giải Kí hiệu ( ) ( ) 1 2 O , O lần lượt là đường tròn đường kính AA′ và BB′ . Gọi M là trung điểm AB. Dễ thấy (M,MA)∩(O1) = {A,E},(M,MA)∩(O2 ) = {B,F} (E, F lần lượt là chân đường cao hạ từ A, B của tam giác ABC). Do AE, BF giao nhau tại trực tâm H và áp dụng định lý về tâm đẳng phương ta suy ra trục đẳng phương của (O1) và (O2 ) đi qua H (đpcm). Bài tập 3.7 Cho đường tròn tâm O đường kính AB . Một điểm H thuộc đoạn AB. Đường thẳng qua H vuông góc với AB cắt đường tròn tại C. Đường tròn đường kính CH cắt AC, BC và (O) lần lượt tại D, E và F. a) Chứng minh rằng AB, DE và CF đồng quy. b) Đường tròn tâm C bán kính CH cắt (O) tại P và Q. Chứng minh rằng P, D, E, Q thẳng hàng. Lời giải a) Ta có 2 Trang 26 P Q M E D C A O H B CA.CD = CH = CB.CE , suy ra ADEB nội tiếp. Xét các đường tròn (ADEB), (O) và đường tròn đường kính CH, thì DE, AB và CF lần lượt là các trục đẳng phương của các cặp đường tròn trên nên chúng đồng quy. b) Ta có PQ là trục đẳng phương của
  • 28. (C) và (O) nên OC ⊥ PQ . Ta cũng dễ thấy OD ⊥ DE . Hơn nữa M chính là tâm đẳng phương của ba đường tròn (O), ( C) và đường tròn đường kính CH. Suy ra PQ đi qua M. Vậy DE, PQ cùng đi qua M và cùng vuông góc với OC nên trùng nhau. Hay D, E, P, Q thẳng hàng. Trang 27
  • 29. Trang 28 PHẦN C: BÀI TẬP ĐỀ NGHỊ 1. ĐỀ BÀI Bài tập 1 Cho tam giác ABC, trực tâm H. M 1, M 2, M 3 lần lượt là trung điểm BC, CA, AB. (M 1, M 1H) ∩ BC={A 1, A 2}, (M 2, M 2H) ∩ AC={B 1, B 2}, (M 3, M 3H) ∩ AB={C 1, C 2}, CMR: A 1, A 2 B 1, B 2, C 1, C 2 cùng thuộc một đường tròn. Bài tập 2 Cho hình thang ABCD (ABCD). K, L là hai điểm trên AB, CD sao cho AK BK = DL CL . Giả sử P, Q nằm trên đoạn thẳng KL sao cho ∠APB =∠BCD và ∠CQD=∠ABC. CMR bốn điểm P, Q, B, C cùng thuộc một đường tròn. Bài tập 3 Cho tứ giác ABCD có cạnh AB và CD cắt nhau tại I, AD và BC cắt nhau tại K. a) CMR trực tâm của các tam giác AID, ABK, BCI, CDK thẳng hàng. b) CMR trung điểm của các đoạn thẳng AC, BD, IK thẳng hàng. Bài tập 4 Cho tam giác ABC có các đường phân giác (trong và ngoài) tại các đỉnh A, B, C theo thứ tự cắt các cạnh đối tại D và D’, E và E’, K và K’. Chứng minh rằng trung điểm của các đoạn thẳng DD’, EE’, KK’ thẳng hàng. Bài tập 5 Cho tứ giác lồi ABCD. Qua đỉnh D của tứ giác kẻ các đường thẳng a, b, c sao cho a ⊥ DA, b⊥ DB, c⊥ DC. Các đường thẳng a, b, c này theo thứ tự cắt các đường thẳng BC, CA, AB tại A’, B’, C’. CMR ba điểm A’, B’, C’ thẳng hàng. Bài tập 6 Cho tam giác ABC. Các phân giác ngoài góc A, B, C lần lượt cắt cạnh đối diện tại A1, B1, C1. CMR A1, B1, C1 thẳng hàng và nằm trên đường vuông góc với đường thẳng nối tâm đường tròn nội tiếp và tâm đường tròn ngoại tiếp tam giác ABC. Bài tập 7
  • 30. Cho tam giác ABC nội tiếp (O). (I), (Ia) lần lượt là đường tròn nội tiếp và bàng tiếp góc A. Giả sử I Ia giao BC và (O) lần lượt tại A’, M. Gọi N là trung điểm cung MBA. NI, NIa giao (O) lần lượt tại S,T. CMR S, T, A’ thẳng hàng. Bài tập 8 (Định lý Brianchon) Cho lục giác ABCDEF ngoại tiếp (O). CMR AD, BE, CF đồng quy. Bài tập 9 Cho tam giác ABC và đường thẳng d. Gọi A’, B’, C’ lần lượt là hình chiếu của A, B, C trên d. Gọi d 1, d 2, d 3 theo thứ tự là các đường thẳng đi qua A’, B’, C’ và vuông góc với Trang 29 BC, CA, AB. Chứng minh rằng các đường thẳng d 1, d 2, d 3 đồng quy. Bài tập 10 Cho (O,R) và hai điểm P,Q cố định (P nằm ngoài (O), Q nằm trong (O)). Dây cung AB của (O) luôn đi qua Q. PA, PB lần lượt giao (O) lần thứ hai tại D,C. Chứng minh rằng CD luôn đi qua một điểm cố định. Bài tập 11 Cho (O1,R1) tiếp xúc ngoài với (O2,R2) tại M (R2R1). Xét điểm A di động trên đường tròn sao cho A,O1,O2 không thẳng hàng. Từ A kẻ tiếp tuyến AB, AC tới (O 1). Các đường thẳng MB, MC cắt tại (O 2) tại E, F. D là giao điểm của EF với tiếp tuyến tại A của (O 2). Chứng minh rằng D di động trên một đường thẳng cố định. Bài tập 12 Cho tam giác ABC có D là trung điểm của cạnh BC. Gọi d là đường thẳng qua D và vuông góc với đường thẳng AD. Trên đường thẳng d lấy một điểm M bất kì. Gọi E, F lần lượt là trung điểm của các đoạn thẳng MB, MC. Đường thẳng qua E vuông góc với d cắt đường thẳng AB tại P, đường thẳng qua F vuông góc với d cắt đường thẳng AC tại Q. Chứng minh rằng đường thẳng qua M, vuông góc với đường thẳng PQ luôn đi qua một điểm cố định khi M di động trên đường thẳng d.
  • 31. Trang 30 2. LỜI GIẢI Bài tập 1 Cho tam giác ABC, trực tâm H. M 1, M 2, M 3 lần lượt là trung điểm BC, CA, AB. (M 1, M 1H) ∩ BC={A 1, A 2}, (M 2, M 2H) ∩ AC={B 1, B 2}, (M 3, M 3H) ∩ AB={C 1, C 2}. CMR: A 1, A 2, B 1, B 2, C 1, C 2 cùng thuộc một đường tròn. Lời giải Do M 1M 2 // AB và AB⊥HC nên M 1M 2⊥HC Suy ra HC là trục đẳng phương của (M 1) và (M 2). ⇒ CA 1 . CA 2 = CB 1 . CB 2 Suy ra A 1, A 2, B 1, B 2 thuộc đường tròn (W 1). Tương tự A 1, A 2,C 1, C 2 thuộc đường tròn (W 2), C 1, C 2, B 1, B 2 thuộc đường tròn (W 3). Nếu 6 điểm A 1, A 2, B 1, B 2, C 1, C 2 không cùng thuộc một đường tròn thì các trục đẳng phương của 3 đường tròn (W 1), (W 2), (W 3) phải đồng quy tại một điểm, nhưng chúng lại cắt nhau tại A, B, C nên vô lý. Vậy ta có đpcm. Bài tập 2 Cho hình thang ABCD (ABCD). K, L là hai điểm trên AB, CD sao cho AK BK = DL CL . Giả sử P, Q nằm trên đoạn thẳng KL sao cho ∠APB =∠BCD và ∠CQD=∠ABC. CMR bốn điểm P, Q, B, C cùng thuộc một đường tròn. Lời giải Từ giả thiết, AK BK = DL CL suy ra AD, BC, KL đồng quy tại E. Dựng đường tròn (O1) đi qua hai điểm C, D và tiếp xúc với BC, (O2) đi qua hai điểm A; B và tiếp xúc với BC. Khi đó ∠DQC = ∠ABC=∠DCE nên Q∈(O1), tương tự P∈(O2).
  • 32. Trang 31 Gọi F là giao điểm thứ hai của EQ với (O1). Ta có: EF . EQ = EC 2 (1) Mặt khác, dễ dàng có ∠O1CD=∠O2BA do đó △ AO2B ~ △ DO 1C ⇒ O 1C O 2B = DC AB = EC EB =k ⇒ E, O 1, O 2 thẳng hàng và EO 1 EO 2 =k ⇒ → EO 1 = k → EO 2 Suy ra phép vị tự H(E,k): (O1)→ (O2). Mà E, F, P thẳng hàng, F∈ (O1), P∈ (O2) nên → EF =k → EP ⇒ EF EP =k= EC EB (2) Từ (1), (2) suy ra EP . EQ = EC . EB Vậy 4 điểm P, Q, B, C cùng thuộc một đường tròn (đpcm). Bài tập 3 Cho tứ giác ABCD có cạnh AB và CD cắt nhau tại I, AD và BC cắt nhau tại K. a) CMR trực tâm của các tam giác AID, ABK, BCI, CDK thẳng hàng. b) CMR trung điểm của các đoạn thẳng AC, BD, IK thẳng hàng. Lời giải a) Gọi O 1, O 2, O3 theo thứ tự là trung điểm của AC, BD, IK. Các đường cao của tam giác CDK lần lượt là CC’, DD’, KK’ và H là trực tâm. Kí hiệu Γ 1, Γ 2, Γ 3 theo thứ tự là các đường tròn tâm O 1 bán kính O 1A, tâm O 2 bán kính O 2B, tâm O 3 bán kính O 3K. Ta có phương tích của điểm H đối với đường tròn Γ 1 và Γ 2 là P (H/Γ 1)= HC . HC’ P (H/Γ 2) = HD . HD’ . Xét đường tròn đường kính CD có HC . HC’ = HD . HD’ Do đó H thuộc trục đẳng phương của hai đường tròn Γ 1 và Γ 2.
  • 33. Chứng minh tương tự, trực tâm các tam giác AID, ABK, BCI cũng có cùng phương tích đối với hai đường tròn Γ 1 và Γ 2. Vậy trực tâm của các tam giác AID, ABK, BCI, CDK Trang 32 thẳng hàng ( điều cần chứng minh) b) O 3 là trung điểm IK, ta cũng chứng minh được P (H/Γ 2) = P (H/Γ 3). Xét trực tâm J của tam giác BCI, khi đó P (J/Γ 1)= P (J/Γ 2)= P (J/Γ 3) suy ra ba đường tròn này đôi một nhận đường thẳng HJ làm trục đẳng phương, vì vậy O 1, O 2 và O 3 thẳng hàng (đpcm) . Lưu ý: HJ⊥ O 1O 3. Bài tập 4 Cho tam giác ABC có các đường phân giác (trong và ngoài) tại các đỉnh A, B, C theo thứ tự cắt các cạnh đối tại D và D’, E và E’, K và K’. Chứng minh rằng trung điểm của các đoạn thẳng DD’, EE’, KK’ thẳng hàng.
  • 34. Trang 33 Lời giải Gọi O 1, O 2, O 3 theo thứ tự là trung điểm DD’, EE’, KK’ ; gọi Γ 1, Γ 2, Γ 3 theo thứ tự là các đường tròn tâm O 1 bán kính O 1D, tâm O 2 bán kính O 2E, tâm O 3 bán kính O 3K. Giả sử Γ là đường tròn ngoại tiếp tam giác ABC có tâm O bán kính R. Vì ΔO 1BA ∽ ΔO 1AC nên O 1B.O 1C = O 1A2 = O 1D2 , từ đó: P (O 1/Γ) = O 1O2 - R2 = O 1B . O 1C = O 1D2 Nhưng P (O/Γ 1) = OO 2 - O 1 1D2 , suy ra: P (O/Γ 1) = R2 Chứng minh tương tự ta có: P (O/Γ 2) = P (O/Γ 3) = R2 Mặt khác nếu gọi H là trực tâm tam giác DEK, tương tự như ví dụ 3 ta chứng minh được: P (H/Γ 1) = P (H/Γ 2) = P (H/Γ 3) Vậy ba đường tròn Γ 1, Γ 2, Γ 3 đôi một nhận đường thẳng OH làm trục đẳng phương do đó O 1, O 2, O 3 thẳng hàng. Bài tập 5 Cho tứ giác lồi ABCD. Qua đỉnh D của tứ giác kẻ các đường thẳng a, b, c sao cho a ⊥ DA, b⊥ DB, c⊥ DC. Các đường thẳng a, b, c này theo thứ tự cắt các đường thẳng BC, CA, AB tại A’, B’, C’. CMR ba điểm A’, B’, C’ thẳng hàng. Lời giải Gọi trung điểm của AA’, BB’, CC’ theo thứ tự là O 1, O 2, O 3 ; gọi Γ 1, Γ 2, Γ 3 theo thứ tự là các đường tròn tâm O 1 bán kính O 1A, tâm O 2 bán kính O 2B, tâm O 3 bán kính O 3C, thế thì P (D/Γ 1) = P (D/Γ 2) = P (D/Γ 3) = 0.
  • 35. Trang 34 Mặt khác, nếu gọi H là trực tâm tam giác ABC thì tương tự ví du 3 ta có P (H/Γ 1) = P (H/Γ 2) = P (H/Γ 3) . Từ đó ba điểm O 1, O 2, O 3 thẳng hàng. Gọi P, Q, R theo thứ tự là trung điểm của BC, CA, AB. Sử dụng định lí Menelaus cho tam giác PQR và ba điểm thẳng hàng O 1, O 2, O 3, ta có: O 1Q O 1R . O 2R O 2P . O 3P O 3Q = 1. Nhưng O 1Q O 1R = A’C A’B ; O 2R O 2P = B’A B’C ; O 3P O 3Q = C’B C’A . Áp dụng định lí Menelaus đảo cho tam giác ABC, ta có A’, B’, C’ thẳng hàng. Bài tập 6 Cho tam giác ABC. Các phân giác ngoài góc A, B, C lần lượt cắt cạnh đối diện tại A1, B1, C1. CMR A1, B1, C1 thẳng hàng và nằm trên đường vuông góc với đường thẳng nối tâm đường tròn nội tiếp và tâm đường tròn ngoại tiếp tam giác ABC. Lời giải Gọi A2B2C2 là tam giác tạo bởi 3 phân giác ngoài góc A, B, C. Dễ dàng có AA2⊥ B2C2 , BB2⊥ A2C2 , CC2⊥ A2B2 . Tứ giác BC2B2C nội tiếp nên A1C2 . A 1B 2 = A1B . A1C Tương tự B1C2 . B1A 2 = B1A . B1C , C1B2 . C1A2 = C1A . C1B Suy ra A1,B1,C1 cùng nằm trên trục đẳng phương của đường tròn (O) ngoại tiếp tam giác ABC và đường tròn (J) ngoại tiếp tam giác A2B2C2. Mà (O) là đường tròn Ơ-le của tam
  • 36. giác A2B2C2, AA2, BB2 ,CC2 giao nhau tại trực tâm I của tam giác A2B2C2 (cũng đồng thời là tâm đường tròn nội tiếp tam giác ABC) suy ra I, O, J thẳng hàng. Vậy đường thẳng qua A1, B1, C1 vuông góc với OI (đpcm) Bài tập 7 Cho tam giác ABC nội tiếp (O). (I), (Ia) lần lượt là đường tròn nội tiếp và bàng tiếp góc A. Giả sử I Ia giao BC và (O) lần lượt tại A’, M. Gọi N là trung điểm cung MBA. NI, NIa giao (O) lần lượt tại S,T. CMR S, T, A’ thẳng hàng. Lời giải Trang 35 Ta có ∠NTS= 1 2 (sđ ΝΑ+ sđ S Α )= 1 2 (sđΝΜ +sđ AS )=∠NIM ⇒∠IaTS=∠IaIS Suy ra tứ giác IaTIS nội tiếp (w1) Mặt khác,∠ IBIa=∠ ICIa=90° nên tứ giác IBIaC nội tiếp (w2) Ta thấy I Ia là trục đẳng phương của (w1) và (w2), BC là trục đẳng phương của (O) và (w2), TS là trục đẳng phương của (O) và (w1) Theo định lý về tâm đẳng phương thì I Ia, TS, BC đồng quy tại A’. Vậy T, A’, S thẳng hàng (đpcm) Bài tập 8 (Định lý Brianchon) Cho lục giác ABCDEF ngoại tiếp (O). CMR AD, BE, CF đồng quy. Lời giải Gọi G, H, I, J, K, L lần lượt là tiếp điểm của AB, BC, CD, DE, EF, FA với (O). Trên tia KF,HB, GB, JD, ID, LF lần lượt lấy các điểm P, S, Q, R, N, M sao cho KP = SH = GQ = JR = IN = LM.
  • 37. Dựng (O1) tiếp xúc với EF,CB tại P, S, (O2) tiếp xúc AF, CD tại M, N, (O3) tiếp xúc AB, ED tại Q, R. Ta có FP=PK-FK=LM-LF=FM, CS=SH+HC=IN+IC=CN Suy ra FC là trục đẳng phương của (O1) và (O2). Tương tự AD là trục đẳng phương của (O2) và (O3), BE là trục đẳng phương của (O3) và (O1). Áp dụng định lý về tâm đẳng phương ta có AD, BE, CF đồng quy (đpcm). Ta chứng minh ba đường thẳng đó là các trục đẳng phương của từng cặp hai đường tròn có tâm không thẳng hàng. Bài tập 9 Cho tam giác ABC và đường thẳng d. Gọi A’, B’, C’ lần lượt là hình chiếu của A, B, C trên d. Gọi d 1, d 2, d 3 theo thứ tự là các đường thẳng đi qua A’, B’, C’ và vuông góc với Trang 36 BC, CA, AB. Chứng minh rằng các đường thẳng d 1, d 2, d 3 đồng quy. Lời giải
  • 38. Trang 37 Gọi O 1, O 2, O 3 theo thứ tự là trung điểm của BC, CA, AB, ta có O 1B’= O 1C’; O 2C’= O 2 A’; O 3A’= O 3B’. Gọi Γ 1, Γ 2, Γ 3 theo thứ tự là các đường tròn tâm O 1 bán kính O 1C’, tâm O 2 bán kính O 2A’, tâm O 3 bán kính O 3B’. Ta có d 3 là trục đẳng phương của Γ 1 và Γ 2. Tương tự d 2 là trục đẳng phương của Γ 3 và Γ 1; d 1 là trục đẳng phương của Γ 2 và Γ 3. Vậy, d 1, d 2, d 3 đồng quy. Bài tập 10 Cho (O,R) và hai điểm P,Q cố định (P nằm ngoài (O), Q nằm trong (O)). Dây cung AB của (O) luôn đi qua Q. PA, PB lần lượt giao (O) lần thứ hai tại D,C. Chứng minh rằng CD luôn đi qua một điểm cố định. Lời giải Gọi E là giao điểm thứ hai khác P của PQ với đường tròn ngoại tiếp tam giác PAB. CD giao PQ tại F. Ta có OQ2 - R2 = QA . QB = QP . QE , mà P,Q cố định nên QP = const, suy ra QE = const, do đó E cố định. Mặt khác∠PDC=∠PBA=∠PEA nên tứ giác DAEF nội tiếp. Suy ra PO2 - R2 = PD . PA = PE . PF . Do P,E cố định nên PE =const, suy ra PF =const. Do đó F cố định. Vậy CD luôn đi qua điểm F cố định (đpcm) Bài tập 11 Cho (O1,R1) tiếp xúc ngoài với (O2,R2) tại M (R2R1). Xét điểm A di động trên đường tròn sao cho A,O1,O2 không thẳng hàng. Từ A kẻ tiếp tuyến AB, AC tới (O 1). Các đường thẳng MB, MC cắt tại (O 2) tại E, F. D là giao điểm của EF với tiếp tuyến tại A của (O 2). Chứng minh rằng D di động trên một đường thẳng cố định. Lời giải
  • 39. Trang 38 Qua M kẻ tiếp tuyến chungcủa (O 1) và (O 2). Ta có ∠MCA= ∠CMy=∠FMD=∠FAM Do đó, △FAM~△FCA (g.g) ⇒ FA2 = FM . FC = FO 2 - R 1 2 (1) 1 Tương tự, EA2 = EO 2 - R 1 2 (2) 1 Coi (A,0) là đường tròn tâm A, bán kính 0 thì từ (1),(2), ta được EF là trục đẳng phương của (A,0) với (O 1). Mà D nằm tren EF nên DA2 = DO 2 - R 1 2 1 ⇒ P D/(O 1)=P D/(O 2) Vậy D nằm trên trục đẳng phương của hai đường tròn cố định (O 1) và (O 2). Bài tập 12 Cho tam giác ABC có D là trung điểm của cạnh BC. Gọi d là đường thẳng qua D và vuông góc với đường thẳng AD. Trên đường thẳng d lấy một điểm M bất kì. Gọi E, F lần lượt là trung điểm của các đoạn thẳng MB, MC. Đường thẳng qua E vuông góc với d cắt đường thẳng AB tại P, đường thẳng qua F vuông góc với d cắt đường thẳng AC tại Q. Chứng minh rằng đường thẳng qua M, vuông góc với đường thẳng PQ luôn đi qua một điểm cố định khi M di động trên đường thẳng d. Lời giải d d' I Q P H' K' K H E F D A B C M
  • 40. Gọi H, K lần lượt là hình chiếu của B, C lên đường thẳng d. Do D là trung điểm của BC nên DH = DK, suy ra AD là trung trực của HK⇒AH =AK. Gọi (ω) là đường tròn tâm A đi qua H và K. Gọi H’, K’ lần lượt là các điểm đối xứng với H, K qua các đường thẳng AB, AC ⇒H’, K’ thuộc (ω) . Giả sử các đường thẳng HH’, KK’ cắt nhau tại I thì I là điểm cố định (*) Ta có : PE // BH (cùng vuông góc với d) mà PE đi qua trung điểm của MB nên cũng qua trung điểm của MH⇒PE là trung trực của MH ⇒ PH = PM. Gọi 1 (ω ) là đường tròn tâm P đi qua H và M, do tính đối xứng nên H’ cũng thuộc 1 (ω ) . Hoàn toàn tương tự, ta cũng có: QF là trung trực của MK; nếu gọi 2 (ω ) là đường tròn tâm Q đi qua K và M thì K’thuộc 2 (ω ) . Ta lại có: +(ω) , 1 (ω ) cắt nhau tai H, H’ nên HH’ là trục đẳng phương của (ω) , 1 (ω ) . +(ω) , 2 (ω ) cắt nhau tai K, K’ nên KK’ là trục đẳng phương của (ω) , 2 (ω ) . Mặt khác : M cùng thuộc 1 (ω ) , 2 (ω ) và P, Q lần lượt là tâm của 1 (ω ) , 2 (ω ) nên đường thẳng d’ qua M, vuông góc với PQ chính là trục đẳng phương của 1 (ω ) , 2 (ω ) . Từ đó suy ra: HH’, KK’, d’ đồng quy tại tâm đẳng phương của ba đường tròn (ω) , 1 (ω ) , 2 (ω ) (**) Từ (*) và (**) suy ra d’ đi qua I là điểm cố định. Vậy đường thẳng qua M, vuông góc với đường thẳng PQ luôn đi qua một điểm cố định khi M di động trên đường thẳng d. Trang 39
  • 41. Trang 40 TÀI LIỆU THAM KHẢO 1. Đoàn Quỳnh, Văn Như Cương, Trần Nam Dũng, Nguyễn Minh Hà, ĐỗThanh Sơn, Lê Bá Khánh Trình: Tài liệu giáo khoa chuyên Toán – Hình học 10. 2. Một số tài liệu trên Internet. 3. Crux Mathematicorum with Mathematical Mayhem. 4. Dusan Djukic, Vladimir Jankovic, Ivan Matic, Nikola Petrovic: The IMO Compedium. A collection of Problems Suggested for the International Mathematical Olympiads 1959-2004. 5. Mathematical Reflection. 6. Website: http://www.imo-official.org/ 7. Website: http://www.artofproblemsolving.com/Forum/portal.php?ml=1